New Free 120 NBME Step 1 Explanations [February 2024]

Author image
Reviewed by
Dr. Anthony Roviso
Key Takeaway

The NBME updated the Free 120 Step 1 questions in February 2024. Below are explanations for every question in Blocks 1-3.

If you're studying for USMLE Step 1, check out Med School Bootcamp. We have the best lesson videos focused on the high-yield concepts that show up on Step 1, Anki-style quiz questions, and a representative Step 1 question bank to make learning medicine easy.

Table of Contents

    Block 1

    1. Correct Answer: (A) This patient has developed slipped capital femoral epiphysis (SCFE), as evidenced by knee pain, anterior hip tenderness, and reduced hip range of motion on internal rotation, abduction, and flexion. SCFE is most common in adolescent males (i.e., ages 10-16) and is commonly associated with obesity. This patient has exam findings of obesity (BMI > 30) and insulin resistance, including hyperpigmented, thickened skin observed at the nape of his neck (i.e., acanthosis nigricans).

      Incorrect Answers: (B) This patient’s mother has type 2 diabetes mellitus, and his maternal grandmother has osteoporosis, neither of which are known risk factors for SCFE. (C) Medication use, especially corticosteroids, is associated with avascular necrosis. This is a potential complication of SCFE but not a risk factor. (D) While current fractures or trauma can increase the risk of SCFE, previous fractures do not. (E) Physical activity is associated with potential overuse myalgia and arthralgia, but it is unlikely to cause SCFE.
    2. Correct Answer: (D) Keratosis pilaris (KP) is a common dermatologic condition characterized by keratinized hair follicles that cause nonpruritic and nonpainful erythematous bumps, most commonly on the extensor surfaces of the arms and thighs. While some patients consider it a cosmetic complaint or a source of social embarrassment, KP is a benign condition. There is typically no need for treatment, but keratolytic medications may be prescribed if patients develop worry or social impairment. For additional information, check out Med School Bootcamp's video on Dermatologic Histopathology.

      Incorrect Answers: (A) Patients with eczema (i.e., atopic dermatitis) commonly present with erythematous, pruritic patches in the flexural distributions of the elbows and knees. (B) Folliculitis is an infection of the hair follicles that typically manifests as clusters of tender and erythematous pustules that can occur anywhere hair is found. (C) Hidradenitis suppurativa (HS) is a chronic inflammatory dermatologic disorder characterized by painful nodules and draining sinus tracts in the intertriginous areas. (E) Urticaria manifests as erythematous, edematous, and pruritic plaques that can occur anywhere on the body and often resolve spontaneously within 2-3 hours.
    3. Correct Answer: (B) This patient with a known history of obstructive sleep apnea (OSA) and intermittent use of a continuous positive airway pressure (CPAP) device has developed increased daytime somnolence, signs of cyanosis (e.g., blue lips, earlobes, and nail beds), and respiratory acidosis on arterial blood gas analysis (i.e., elevated PCO2, decreased PO2, and acidemia). OSA is characterized by recurrent episodes of partial or complete collapse of the upper airway during sleep, which can lead to inadequate sleep quality and chronic intermittent hypoxia. Moreover, recurrent hypoxic conditions during sleep can stimulate hypoxia-inducible factor-2 (i.e., the principal regulator of erythropoietin release), leading to secondary polycythemia. The resulting increase in hemoglobin, hematocrit, and erythrocyte mass is the body’s attempt to compensate for chronic oxygen desaturation.

      Incorrect Answers: (A) Serum bicarbonate is most likely to be increased in OSA as a renal compensation for carbon dioxide retention from chronic respiratory acidosis. (C) Total lung capacity is most likely decreased in a patient with a BMI of 63 kg/m2 due to musculoskeletal restriction and increased abdominal adiposity. (D) OSA is a likely cause of group 3 pulmonary hypertension (chronic-lung disease etiology) that can lead to right ventricular hypertrophy (i.e., cor pulmonale). For additional information, check out Med School Bootcamp's video on Sleep Apnea.
    4. Correct Answer: (A) This patient presents with hemoptysis, shortness of breath, lower extremity edema, and dark urine with the presence of RBCs (i.e., hematuria). He is hypertensive and hypoxemic with diffuse inspiratory crackles on auscultation. Lab results support a diagnosis of acute kidney injury and hematuria with dysmorphic urinary RBCs and rare casts. Biopsy reveals crescentic (i.e., rapidly progressive) glomerulonephritis and linear IgG deposition along the glomerular capillaries. This presentation is pathognomonic for Goodpasture syndrome, a cause of nephritic syndrome and proliferative glomerulonephritis resulting from the formation of antibasement membrane antibodies directed at type IV collagen in the renal and pulmonary capillaries. For additional information, check out Med School Bootcamp's video on Rapidly Progressive Glomerulonephritis.

      Incorrect Answers: (B & C) Lupus nephritis occurs when antibodies against double-stranded DNA (i.e., anti-dsDNA) or nuclear antigens (e.g., anti-Sm, anti-RNP) cross-react with the glomerular basement membrane. This patient does not mention a previous diagnosis of lupus and lacks classic findings suggestive of the disease (e.g., butterfly rash, photosensitivity, oral ulcers, arthralgias). (D) Antiphospholipid antibody syndrome (APS) is a systemic autoimmune disease caused by circulating antibodies directed against phospholipids, often leading to thrombosis and pregnancy complications. Renal involvement typically manifests as renal vessel thrombosis or stenosis. This patient’s history and renal evaluation are not consistent with this condition. (E) Antibodies targeting proteins in neutrophil cytoplasm are associated with several autoimmune conditions involving small vessels that can lead to ANCA-associated glomerulonephritis (AGN). The hallmark lesion in AGN is necrotizing or crescentic glomerulonephritis without deposition of immunoglobulins; this was not observed in this patient.
    5. Correct Answer: (D) This patient's large U-shaped kidney suggests an abnormality resulting from the fusion of the kidneys. The metanephros is the embryonic structure from which the adult kidney develops. Fusion of the inferior poles of the metanephros during ascent can result in the formation of a horseshoe kidney with a classic “U-shaped” appearance. This fusion can also lead to anomalies in the urinary tract, making the patient more prone to urinary tract infections.

      Incorrect Answers: (A & B) While disruptions in the rotation and ascent of the kidneys are associated with a horseshoe kidney, these mechanisms alone would not result in the fusion of the inferior poles and subsequent development of a singular U-shaped kidney. Renal orientation and positioning would be affected instead. (C) Failure of one ureteric bud to develop can lead to renal agenesis, which manifests as the absence of one or both kidneys. The presence of a large, U-shaped kidney is more likely to indicate renal fusion than agenesis.
    6. Correct Answer: (E) When new medications are added to the treatment regimen for patients taking warfarin, adjusting the warfarin dose is necessary to avoid potential complications related to inadequate anticoagulation. This patient’s coagulation study results were within normal range both before and after changes in the warfarin dose, which is the main goal after the addition of ciprofloxacin and rifampin to this patient’s treatment regimen. Rifampin is known to induce hepatic enzymes, specifically cytochrome P450 enzymes, which play a significant role in the metabolism of various drugs, including warfarin. When rifampin induces cytochrome P450 enzymes, warfarin metabolism is accelerated, thereby reducing its plasma levels and anticoagulant effect. Doubling the warfarin dose helps compensate for the increased metabolism caused by rifampin, thereby ensuring that therapeutic anticoagulation levels are maintained and preventing the risk of blood clots.

      Incorrect Answers: (A) The binding of a drug to plasma proteins is a major determinant of drug disposition and can be influenced by several factors. When this occurs, the bound drug is unable to interact with receptors in target tissues; only the free drug interacts with receptors and can produce therapeutic effects. If protein binding is decreased, this would lead to a subsequent increase in free warfarin levels and an increase in INR, which was not observed in this patient. (B) Gut flora imbalances can occur as a result of antibiotic intake and can cause changes in coagulation studies. This patient does not present with signs or symptoms of gut flora eradication (e.g., gastrointestinal distress), making this an unlikely cause of this patient’s laboratory findings. (C & D) Lifestyle and dietary habits can affect warfarin metabolism and coagulation studies. This patient’s lab results were within normal range both on his original coagulation studies and after his warfarin dose was doubled, which suggests that an acute mechanism (e.g., the interaction with a new medication) was involved. If his lab results were abnormal due to lifestyle factors (e.g., healthy diet or chronic alcohol consumption), his baseline labs would have been abnormal, which was not the case.
    7. Correct Answer: (C) This patient presents with 2 weeks of a generalized prodrome, including fever, exudative pharyngitis, tachycardia, hypotension, diffuse lymphadenopathy, anemia, leukopenia, and thrombocytopenia in the absence of definitive lab testing. This is consistent with a primary HIV infection, which commonly manifests with a mononucleosis-like syndrome (e.g., fever, fatigue, myalgia, headache, generalized lymphadenopathy, and oropharyngeal symptoms).

      Incorrect Answers: (A) This is unlikely to be EBV because the heterophile antibody test was negative. (B) Patients with gonococcal pharyngitis are likely to present with a sore throat, pharyngeal exudates, and cervical lymphadenitis consistent with localized infection; diffuse lymphadenopathy would be less likely. (D) Lymphogranuloma venereum (LGV) is caused by serotypes L1-L3 of Chlamydia trachomatis. It is most common in tropical countries, with increased incidence in men who have sexual contact with men. Primary infection results in the formation of small, painless genital ulcerations that heal spontaneously. This is followed by painful inguinal adenopathy with abscess formation and systemic toxicity. (E) Streptococcal pharyngitis is unlikely to cause cytopenias to this degree and is likely to resolve spontaneously in 2-5 days even if untreated.
    8. Correct Answer: (A) This patient’s presentation of headache, shortness of breath with exertion, and blood pressure of 180/105 mmHg suggests poorly controlled chronic hypertension, which can lead to arteriovenous nicking (AV) and end-organ damage. AV nicking is commonly seen on ophthalmoscopic exams of hypertensive patients. It occurs when a retinal vein is compressed (i.e., "nicked") by a retinal artery at an AV crossing point.

      Incorrect Answers: (B) Ocular melanosis and melanoma can both occur due to malfunctioning melanocytes in the uvea and manifest with vision changes, eye pain, and potentially metastatic spread. Risk factors include atypical cutaneous nevi, history of chronic sun exposure, and fair skin, which were not mentioned in this case. (C) Optic neuritis typically manifests with sudden vision loss, eye pain exacerbated by eye movement, and visual field deficits. It is often associated with multiple sclerosis, which typically occurs in young females and causes varying degrees of neurological deficits. (D) Subscapular cataracts typically manifest with a gradual onset of blurred vision, glare sensitivity, and halos associated with bright lights. Risk factors include aging, diabetes mellitus, and long-term corticosteroid use. (E) Retinal detachment most often manifests with visual floaters, flashes of light, and progressive visual field loss. Risk factors include recent trauma, diabetes, and sickle cell disease.
    9. Correct Answer: (D) This patient has developed focal neurologic deficits in the context of rheumatic mitral valve disease and atrial fibrillation. Her deficits include right-sided weakness, Broca’s aphasia (i.e., inability to speak), right lower facial weakness, and dysphagia. Atrial fibrillation can cause a thromboembolic stroke that involves the left middle cerebral artery, which can in turn affect the motor and sensory cortices as well as Broca’s area within the inferior frontal gyrus. For additional information, check out Med School Bootcamp's video on Types of Strokes.

      Incorrect Answers: (A) The anterior inferior cerebellar artery (AICA) branches from the basilar artery and supplies the anteroinferior portion of the cerebellum. Blockage of this vessel would cause lateral pontine syndrome, potential symptoms of which may include vertigo, hearing loss, and ataxia. (B) The superior cerebellar artery (SCA) branches from the basilar artery and supplies the superior region of the cerebellum. Blockage would cause symptoms such as ataxia, dysmetria, dysarthria, and nystagmus. (C) The posterior cerebral artery supplies the occipital lobe, inferomedial temporal lobe, and midbrain. A blockage in this vessel is associated with contralateral homonymous hemianopia with macular sparing. (E) The anterior communicating arteries supply the corpus callosum and the medial surfaces of the anterior cerebral arteries. A blockage in this location often leads to personality changes, cognitive deficits, and lower limb weakness.
    10. Correct Answer: (A) This patient with a history of gastroesophageal reflux disease (GERD) managed with the maximum dosage of omeprazole notes that his medication stops working during the holidays when his food and alcohol intake is increased. Since this level of specificity with regard to diminished drug efficacy is unusual, the physician should inquire about the severity and quantity of the increased food and alcohol intake. This type of binging behavior is likely worsening the patient’s reflux to a point at which the medication cannot work. Increased gastric distention from overeating can increase intragastric pressure, forcing the lower esophageal sphincter (LES) open and causing reflux. This patient also has a risk of medication-alcohol interactions, as alcohol can lead to LES relaxation and subsequent worsening of reflux.

      Incorrect Answers: (B, C, D, E) While these steps may be necessary, it is most appropriate for the physician to explore the degree of this patient’s increased food and alcohol intake prior to changing his medical management.
    11.  Correct Answer: (C) This patient’s presentation is consistent with pernicious anemia (e.g., weakness, paresthesias, altered mental status, spasticity, loss of vibratory sensation). This diagnosis is further supported by lab findings of antiparietal cell antibodies, methylmalonic acidemia, homocysteinemia, and megaloblastic anemia. A type II hypersensitivity reaction leads to antibody formation against gastric parietal cells and a subsequent decrease in gastric acid and intrinsic factor production. This ultimately results in impaired vitamin B12 absorption in the terminal ileum. Vitamin B12 is required for the conversion of homocysteine to methionine via impaired methionine synthase function. Deficiency leads to homocysteine elevation and methionine deficiency. This conversion also normally recycles tetrahydrofolate. Without these processes, DNA synthesis is impaired, leading to the formation of megaloblastic hematopoietic cells.

      Vitamin B12 is also responsible for the conversion of methylmalonyl CoA to succinyl CoA via methylmalonyl CoA mutase, with deficiency leading to elevations in methylmalonyl CoA (i.e., methylmalonic acid). This is important to note because methylmalonic acid levels can differentiate vitamin B12 deficiency-induced megaloblastic anemia from vitamin B9 (i.e., folate) deficiency-induced megaloblastic anemia. Methylmalonic acid levels are increased in vitamin B12 deficiency and normal in folate deficiency.

      Incorrect Answers: (A, B, D, E) Impairments in the synthesis of cysteine, glutamine, phenylalanine, and tyrosine are associated with a broad range of metabolic and symptomatic disturbances (e.g., motor dysregulation, neurological dysfunction, and laboratory abnormalities). However, only methionine is directly involved in the pathway that would lead to increased methylmalonic acid and total homocysteine levels. For more information, check out Med School Bootcamp's video on Protein Metabolism.
    12. Correct Answer: (B) This patient with a history of distant right colon resection and chemotherapy presents with fatigue on follow-up examination. CT imaging reveals multiple new hepatic and pulmonary masses consistent with cancer recurrence and metastasis. This patient’s primary gastrointestinal malignancy most likely traveled through the right-sided mesenteric venules into the superior mesenteric vein and portal vein, through the liver (depositing hepatic metastases) and inferior vena cava, and, finally, into the lungs. For additional information, check out Med School Bootcamp's video on Veins, Anastomoses, and Hemorrhoids.

      Incorrect Answers: (A, C, D, E, F, G) These answer choices are incorrect because none of these vessels are associated with the most direct route of blood flow from the right colon to the lungs.
    13. Correct Answer: (F) This patient presents with increased urinary frequency and excessive thirst. His urine osmolality is 50 mOsmol/kg. After administration of ADH, his urine osmolality is within the reference range (50-1200 mOsmol/kg), indicating that he was likely experiencing central diabetes insipidus (DI) due to impaired secretion of ADH, which is produced by the supraoptic nucleus of the hypothalamus. DI is divided into two subtypes based on the patient's urine osmolality response after the administration of desmopressin (i.e., the synthetic form of ADH): 

      - Central DI:
      ADH administration leads to an increase in urine osmolality, as seen in this patient. 
      - Nephrogenic DI:
      ADH administration results in little to no response, and low urine osmolality continues.

      For additional information, check out Med School Bootcamp's video on Sodium.

      Incorrect Answers: (A, B, C, D, E, G) This patient’s symptoms are a result of decreased secretion of ADH. None of these answer choices are associated with the production of ADH and would not explain this patient’s clinical presentation. 
    14. Correct Answer: (C) This patient's constellation of intermittent headaches, elevated blood pressure (154/100 mmHg), left ventricular hypertrophy (i.e., electrocardiogram findings of left axis deviation and tall R waves in leads V5 and V6), and cardiac exam findings (i.e., point of maximal impulse displaced to the left and occasional skipped beats) is most consistent with hypertrophic cardiomyopathy (HCM). HCM is a genetic cardiac disorder characterized by abnormal thickening of the left ventricular wall, resulting from an increase in the synthesis of contractile filaments (i.e., sarcomeres) within the cardiac muscle cells (i.e., myocytes). The increased contractile filaments cause the left ventricle to become hypertrophied and can lead to various cardiac symptoms, including arrhythmias. Patients may present with symptoms of chest pain, dyspnea, syncope, and headaches. This patient’s headaches were likely caused by hypertension secondary to HCM.

      Incorrect Answers: (A) Excessive accumulation of glycogen in the left ventricle can be seen in glycogen storage diseases (e.g., Pompe disease). This condition manifests in infancy with symptoms of hypotonia, failure to thrive, and cardiomegaly. (B) Fibrosis of intraventricular conduction pathways is associated with symptoms of palpitations or syncope and can lead to the development of cardiac arrhythmias. (D) Misfolding of cytoskeletal proteins could lead to structural damage to the heart but would not account for the left ventricular hypertrophy seen in this patient. (E) Myocyte hyperplasia as a result of the induction of embryonic genes could lead to tumor formation such as a cardiac myxoma. These tumors typically arise from the left atrium and are associated with palpitations, fever, and weight loss.
    15. Correct Answer: (B) This patient has evidence of exocrine pancreatic insufficiency, including fatty stools (i.e., large volume, floating) and significant weight loss. In the setting of insufficient pancreatic enzyme (i.e., lipase) secretion and activation, fat-soluble vitamins (i.e., A, D, E, K) are likely to become deficient. This patient has symptoms of vitamin A deficiency (e.g., xerosis cutis). For additional information, check out Med School Bootcamp's video on Pancreatic Insufficiency.

      Incorrect Answers: (A) Magnesium deficiency is associated with muscle cramps, fatigue, weakness, and cardiac arrhythmias. (C) Vitamin B12 deficiency is associated with macrocytic anemia, peripheral neuropathy, and neuropsychiatric disturbances. (D) Vitamin C deficiency (i.e., scurvy) is classically characterized by fatigue, swollen gums, petechiae, and poor wound healing. (E) Zinc deficiency is associated with delayed wound healing, suppressed immunity, and hypogonadism in males.
    16. Correct Answer: (B) For data that follow a normal distribution, the formula for a 95% confidence interval is x̅ +/- (1.96 x Standard Error), where x̅ is the mean. Therefore, the lower end of the confidence interval is 130 - (1.96 x 5) = 120.2; the higher end of the confidence interval is 130 + (1.96 x 5) = 139.8. This best matches the range of 120-140.

      Incorrect Answers: (A, C, D, E) These answers are incorrect measurements of the range associated with a 95% confidence interval for the findings described in this scenario. For more information, check out Med School Bootcamp’s video on Statistical Distributions.
    17. Correct Answer: (E) This patient presenting with mild respiratory distress, barrel chest, and decreased FEV1:FVC ratio likely has emphysema. Emphysema is a component of chronic obstructive pulmonary disease (COPD) characterized by loss or damage of alveolar elastin and lung parenchyma due to a protease/antiprotease imbalance. This causes increased alveolar collapse on expiration, air trapping and dynamic hyperinflation, impaired ventilation, and increased dead space, leading to hypoxemia and hypercapnia. Patients may experience shortness of breath and dyspnea on exertion and/or at rest. Over time, patients may develop severe hyperinflation and a barrel chest. Breathing through pursed lips is performed in order to prevent expiratory alveolar collapse (auto-PEEP). Pulmonary function testing classically reveals a decreased FEV1:FVC ratio (<70%), increased residual volume (i.e., obstruction), and impaired diffusion (i.e., decreased DLCO). For additional information, check out Med School Bootcamp’s video on COPD.

      Incorrect Answers: (A) Asthma typically manifests in childhood with intermittent episodes of wheezing, coughing, and shortness of breath. This patient’s physical examination findings and history of cigarette smoking are more suggestive of emphysema. (B) Bronchiectasis is ​​often diagnosed after chronic cough and repetitive pulmonary infections that require antibiotics over several years. Symptoms include progressive dyspnea, intermittent wheezing, and mild hemoptysis. (C) Chronic pulmonary fibrosis is a restrictive lung disease that manifests with dyspnea, digital clubbing, and a non-productive cough. Because it is a restrictive lung disease, the FEV1:FVC ratio would be normal or slightly increased. (D) Cystic fibrosis is an autosomal recessive genetic condition that typically manifests in infancy with failure to thrive and recurrent sinopulmonary infections that persist throughout the patient’s life. For this 39-year-old male whose symptoms began only 6 months ago, this diagnosis is unlikely.
    18. Correct Answer: (D) A type I error (i.e., false positive) occurs if an investigator rejects a null hypothesis that is actually true in the population; a type II error (i.e., false negative) occurs if the investigator fails to reject a null hypothesis that is actually false in the population. Power refers to the ability of a study to correctly reject the null hypothesis when the null hypothesis is false. Mathematically, power = 1 - the probability of making a type II error. 

      In this study, increasing the sample size from 500 to 1500 will increase the study’s power. If the probability that the study will correctly reject the null hypothesis increases, then the probability that the study will fail to reject the null hypothesis when the null hypothesis is false will subsequently decrease. In other words, the risk of making a type II error will decrease as power increases.

      Incorrect Answers: (A, B, C, E) For more information, check out Med School Bootcamp’s video on Testing Errors.
    19. Correct Answer: (B) This patient’s abdominal pain and early satiety combined with the histological images of her gastric wall are suggestive of diffuse gastric adenocarcinoma (i.e., stomach cancer). Immunostaining of cytokeratin is used to detect occult carcinoma cells in lymph nodes, which suggest malignancy. In gastric adenocarcinoma, there is often loss or decreased expression of epithelial cadherins, which are molecules responsible for maintaining cell-to-cell adhesion in normal epithelial tissues. The loss of epithelial cadherin function can disrupt cell adhesion and contribute to the invasive and metastatic properties of cancer cells.

      Incorrect Answers: (A, C, D, E, F) Gastric adenocarcinoma is not closely associated with mutations in these structures. For more information on gastric adenocarcinoma, check out Med School Bootcamp’s video on Malignant Gastric Pathology.
    20. Correct Answer: (D) This patient presents with a BMI of 27, a family history of type 2 diabetes, and elevated blood pressure. These findings are indicative of insulin resistance, which is a key feature of metabolic syndrome. In individuals with insulin resistance, the pancreas compensates by producing higher levels of insulin to maintain normal blood glucose levels. Therefore, an increased fasting insulin concentration is expected in this patient compared to a woman of the same age with a normal weight.

      Incorrect Answers: (A) Metabolic syndrome primarily causes increased synthesis, not excretion, of triglycerides and low-density lipoproteins. Insulin resistance promotes lipogenesis and is responsible for elevated cholesterol levels, but it is not directly related to the excretion of cholesterol. (B) Elevated BMI is associated with increased estrone production in adipose tissue. (C) Leptin is produced in adipose tissue and would be increased in individuals with higher BMI. (E) Growth hormone may decrease the uptake of glucose into cells and increase peripheral insulin resistance. However, excessive growth hormone secretion is associated with pituitary tumors or ectopic production (e.g., lung tumors). Furthermore, other manifestations of excessive growth hormone (e.g., acromegaly) would be expected. (F) Any changes in thyroid function would occur secondary to the metabolic disturbances related to metabolic syndrome. Insulin resistance may affect the conversion of T4 to T3 in peripheral tissues, but this would not lead to an isolated increase in thyroid-stimulating hormone.
    21. Correct Answer: (C) During a myocardial infarction, cardiac function is severely compromised. This patient’s ECG reveals ST-segment elevations and new Q waves in leads V4-V6, representing occlusion of the distal left anterior descending (LAD) and left circumflex coronary artery (LCX). This infarcted tissue comprises a large portion of the left ventricle. In an acute setting, ventricular contractility will decrease in an effort to balance oxygen supply and demand. This is seen as a large drop in cardiac output, which leads to increased retention of blood within the left ventricle and an increased end-diastolic volume (EDV). This is represented by a change from W to Z on the graph.

      Incorrect Answers: (A, B, D, E, F, G, H, I) For additional information, check out Med School Bootcamp's video on Cardiac Function and Venous Return Curves.
    22. Correct Answer: (A) Bipolar I disorder is characterized by at least one episode of mania, which is defined as seven or more days of abnormal and persistently elevated, irritable, or expansive mood along with increased goal-directed activity, behavior, or energy that are not attributed to another medical condition, psychotic disorder, or substance abuse. This patient has evidence of psychotic features, including delusions (e.g., married to the president's brother, receiving specific instructions from a higher being) that further support this diagnosis.

      Incorrect Answers: (B) Brief psychotic disorder involves the sudden onset of psychotic behaviors such as delusions, hallucinations, disorganized speech, and/or catatonic behavior that lasts less than one month. (C) Delusional disorder is diagnosed when a person has one or more non-bizarre (i.e., situations that are not real but also not impossible) delusional thoughts for one month or more. There are several subtypes, which include persecutory, jealous, erotomanic, somatic, grandiose, mixed, and unspecified. (D) Psychotic disorder due to a general medical condition can be attributed to various causes (e.g., abnormal thyroid function, autoimmune conditions, degenerative neurocognitive disorders, etc.). However, the lack of significant medical history in this patient suggests this is not the most likely cause of her symptoms. (E) Schizophrenia is often associated with a combination of positive symptoms (e.g., hallucinations, delusions) and negative symptoms (e.g., social withdrawal, blunted affect), as well as cognitive symptoms (e.g., disorganized thinking, impaired executive function) that have been present for at least six months. For additional information, check out Med School Bootcamp's video on Bipolar Disorders.
    23. Correct Answer: (D) Absorbing and understanding complex information is extremely difficult, especially soon after receiving unexpected emotional news. The physician should give the family time to process and understand a small amount of need-to-know information before giving additional details that may otherwise be forgotten.

      Incorrect Answers: (A, B, C, E) For more information about ethical approaches to patient care, check out Med School Bootcamp’s videos on Medical Ethics.
    24. Correct Answer: (B) This patient is experiencing lower urinary tract symptoms, including impaired bladder emptying, urinary hesitancy, and postvoid dribbling, indicating a lower urinary tract obstruction. Physical exam findings of a diffusely enlarged, firm, and nontender prostate are consistent with benign prostatic hyperplasia (BPH). BPH is a very common condition of advancing age. It is due to prostate stimulation by dihydrotestosterone (DHT), which is converted from testosterone by 5 alpha-reductase. This stimulation leads to hyperplasia of the lateral and middle periurethral lobes of the prostate and subsequent symptomatology.

      Incorrect Answers: (A, C, D, E) For additional information, check out Med School Bootcamp's video on Benign Prostatic Hyperplasia.
    25. Correct Answer: (B) This patient presents with a knife wound through his right chest, tachycardia, normotension, tachypnea, and leftward tracheal shift. This is most likely due to a traumatic pneumothorax, which can cause the following physical exam findings:

      - Opposite tracheal shift due to air filling the pleural space and forcing the contents to the other side of the chest
      - Hyperresonance on percussion due to the presence of intrathoracic air
      - Decreased breath sounds on the ipsilateral side due to lung collapse
      - Decreased fremitus on the ipsilateral side. Air is a poor conductor of low sound frequencies, whereas solid or dense tissue increases the transmission of these frequencies. Therefore, a hemithorax full of air will conduct sound less readily than normal heterogeneous lungs (i.e., air and parenchyma).

      Incorrect Answers: (A, C, D, E) A hemothorax may occur by a similar mechanism of injury (i.e., penetrating wound to the chest) and would manifest with dullness to percussion; however, it is ruled out in this case due to the patient’s hemodynamic stability. Increased fremitus can occur in conditions such as pneumonia (i.e., increased pulmonary density and fluid). The absence of infectious symptoms (i.e., fever, cough, shortness of breath) and history of recent trauma to the chest suggest that this patient’s symptoms are due to a pneumothorax. For additional information, check out Med School Bootcamp's video on Pneumothorax.
    26. Correct Answer: (D) Proto-oncogenes are normal genes that can be converted into oncogenes by mutations, leading to the uncontrolled growth and division of cells. The RET proto-oncogene codes for a receptor tyrosine kinase involved in cell signaling; mutations in RET are known to be associated with the development of medullary thyroid carcinoma (MTC), pheochromocytoma, and hyperparathyroidism, all of which are seen in multiple endocrine neoplasia type 2 (MEN2A). This patient's presentation of a neuroendocrine neoplasm of parafollicular cell origin, a palpable mass in the neck, and a history of pheochromocytoma is suggestive of MEN2A (i.e., Sipple syndrome). MEN2A is an autosomal dominant genetic disorder associated with mutations in the RET proto-oncogene.

      Incorrect Answers: (A, B, C, E) Mutations in these genes are not associated with the development of MEN2A.
    27. Correct Answer: (C) This patient's presentation is suggestive of an aortic dissection. Possible symptoms seen in aortic dissection include:

      - Severe upper back pain and hoarseness resulting from compression of the left recurrent laryngeal nerve
      - Ptosis and miosis of the left eye resulting from Horner syndrome due to involvement of the sympathetic chain
      - Anhidrosis of the left side of the forehead (i.e., Horner syndrome)
      - Left hemiplegia resulting from involvement of the brachiocephalic trunk leading to insufficient blood flow to the right side of the brain
      - A decreased left radial pulse, reflecting decreased blood flow to the left upper extremity
      - A blood pressure difference between the right and left arms (i.e., 160/80 mm Hg in the right arm and 100/60 mm Hg in the left arm), as dissection can disrupt blood flow to one of the subclavian arteries
      - A widened mediastinum on the chest x-ray

      In this case, the dissection resulted in the disruption of the sympathetic fibers that travel along the distal aortic arch and led to the neurological and ophthalmological symptoms seen in this patient.

      Incorrect Answers: (A & B) An aortic dissection involving the aorta distal to the left subclavian artery and extending into the left carotid artery may cause symptoms related to compromised blood flow to the left arm (e.g., decreased radial pulse); however, without involvement of the aortic arch, it would not account for the neurological deficits (i.e., Horner syndrome) seen in this patient. (D) A dissection of the proximal aorta extending into the right subclavian artery would not account for the left-sided deficits observed in this patient. (E) A superior sulcus tumor is associated with shoulder or arm pain that radiates down the ulnar aspect of the arm (i.e., C8-T1 dermatomes), Horner syndrome, and atrophy and sensory deficits of the hand muscles. (F) A thrombus in the left carotid artery is associated with sudden weakness or numbness on one side of the body, difficulty speaking or understanding speech, visual disturbances, and confusion. The widened mediastinum and the pressure difference between the right and left arms are more consistent with an aortic dissection.
    28. Correct Answer: (E) This patient with alcohol use disorder presents with hematuria without pain. He has known exposure to naphthylamine and tobacco. Imaging reveals a mass within the renal pelvis; based on his exposure history, this is most likely due to urothelial (i.e., transitional cell) carcinoma. This is the most common malignancy of the bladder, ureter, and renal pelvis. When the renal pelvis is involved, the classic presentation consists of flank pain and painless hematuria throughout micturition.

      Incorrect Answers: (A) Angiomyolipomas are rare benign renal tumors composed of blood vessels, smooth muscles, and adipose tissues and are often found incidentally. They are associated with genetic conditions such as tuberous sclerosis, whose classic findings include hamartomas of the skin, ash-leaf spots, shagreen patches, cardiac rhabdomyoma, and subependymal nodules. (B) Metastatic spread of melanoma most often involves the skin and subcutaneous tissue, the lungs, the liver, the bones, and the brain. Metastatic spread to the kidneys is rare and not likely the cause of this patient’s symptoms. (C) Nephroblastoma (i.e., Wilms tumor) often manifests in early childhood, typically between the ages of 2 and 5 years; common symptoms include a painless abdominal mass, abdominal swelling, and hematuria. (D) Onycocytomas are common benign renal tumors that are often found incidentally in older individuals. Gross evaluation reveals a well-circumscribed tan-colored mass with a stellate central scar, which is not observed in this patient.
    29. Correct Answer: (B) This patient with a past medical history of hypertension presents with tachycardia, tachypnea, hypertension, and shortness of breath after a cross-country flight. Physical exam reveals evidence of a right lower extremity deep venous thrombosis with extension to the popliteal vein (i.e., pain behind the right knee). This is concerning for a pulmonary embolism. Factor V Leiden mutation is a cause of inherited thrombophilia, which results in the inability of activated protein C to inactivate clotting factor V, leading to enhanced thrombin activation and increased risk of thromboembolic events (e.g., deep venous thrombosis, cerebral venous thrombosis, recurrent pregnancy loss).

      Incorrect Answers: (A) Antithrombin III deficiency is most commonly asymptomatic; it is often discovered if a patient develops a thromboembolism while on heparin treatment, since heparin functions by activating antithrombin III. (C) Glanzmann thrombasthenia is a functional platelet disorder that results in microbleeding (e.g., petechiae, purpura). (D) Protein C deficiency is rarer than Factor V Leiden, but both conditions increase the risk of developing thromboembolisms. Factor V Leiden mutation typically manifests with an increased risk of blood clotting, especially in veins, while protein C deficiency often manifests with recurrent venous thrombosis, skin necrosis after warfarin use, and a higher incidence of arterial thrombosis. (E) Von Willebrand disease is the most common inherited bleeding disorder characterized by microbleeding (e.g., petechiae, purpura); in some cases, it is associated with factor VIII deficiency and commonly manifests with a normal platelet count, prolonged bleeding time, and a slightly increased partial thromboplastin time (PTT) in severe cases. For additional information, check out Med School Bootcamp's video on Thrombophilias.
    30. Correct Answer: (D) This patient has developed symptomatic syringomyelia. Syringomyelia typically manifests with progressive neurological symptoms such as weakness, loss of sensation, and pain in the upper limbs in a "cape-like" distribution. It is important to obtain a history of trauma, as a small percentage of patients develop symptomatic syringomyelia as a complication. This is thought to result from traumatic inflammatory obstruction of the central canal of the spinal cord, leading to cavitary dilatation and subsequent compression of cord structures, including:

      - Lateral spinothalamic tract fibers at the anterior white commissure, resulting in loss of pain and temperature sensation
      - Lower motor neuron compression at anterior horns, resulting in bilateral weakness, paresis, and muscle atrophy at the level of syrinx
      - Upper motor neuron compression in the medial portion of the lateral corticospinal tract, resulting in spastic paresis below syrinx
      - Posterior column, resulting in loss of proprioception and vibratory sensation

      Due to the lack of pain perception, patients may develop injuries and chronic skin ulcerations on the hand.

      Incorrect Answers: (A, B, C, E)
      These risk factors are not commonly associated with the development of syringomyelia. For additional information, check out Med School Bootcamp's video on Central Cord Syndromes.
    31. Correct Answer: (D) This young male presents with painful left eye bulging without a history of trauma or medication use. Imaging reveals a small mass involving the ocular muscles, and subsequent biopsy reveals malignant cells with striations. This most likely represents a rhabdomyosarcoma (i.e., a tumor of primitive skeletal muscle cells known as rhabdomyoblasts), the most common malignant orbital tumor in children.

      Incorrect Answers: (A) Neuroblastoma is a tumor of the adrenal medulla that is most commonly diagnosed in children before the age of four. It is associated with abdominal distension, elevated levels of HVA and VMA (i.e., catecholamines and metanephrines) in urine, and a firm, irregular mass that can cross the midline. (B) Pheochromocytoma is the most common tumor of the adrenal medulla in adults and is associated with episodic headaches, sweating, tachycardia, and elevated HVA and VMA in urine. (C) Retinoblastoma is a malignancy of the retina that most commonly occurs in young children and presents with leukocoria (i.e., white pupil reflex), strabismus, eye redness or swelling, poor vision, and eye pain. (E) The most common symptoms of thyroid cancer are neck swelling detected by the patient or a clinician or incidentally detected thyroid nodules on neck imaging; neither was observed in this case.
    32. Correct Answer: (E) This patient with sickle cell disease (SCD) is presenting with symptoms of biliary colic (e.g., postprandial RUQ pain, nausea, lack of fever) in the presence of gallstones confirmed by ultrasound. This is most likely secondary to underlying hemolysis linked to the patient’s SCD. Typical lab findings in hemolysis include decreased haptoglobin, increased LDH, and increased unconjugated bilirubin. The elevation in unconjugated bilirubin results from heme catabolization and can increase the risk of pigmented calcium bilirubinate gallstone formation.

      Incorrect Answers: (A, B, C, D) For additional information, check out Med School Bootcamp's video on Hemolysis.
    33. Correct Answer: (D) For a normal distribution of data, mean (i.e., average) is the most reliable measure of central tendency. In this example, the mean leukocyte count is 7500 cells/mm3. Therefore, approximately 50% of individuals will have leukocyte counts greater than 7500 cells/mm3, and 50% will have leukocyte counts less than 7500 cells/mm3.

      Incorrect Answers: (A, B, C, E) These answers are incorrect calculations of the total leukocyte count range that would be expected for 50% of the population. For additional information, check out Med School Bootcamp’s video on Statistical Distributions.
    34. Correct Answer: (A) This patient presents with signs and symptoms of hypercalcemia, including increased urinary frequency, constipation, and fatigue in the presence of elevated calcium and decreased parathyroid hormone on serum lab studies. This suggests that an extra-endocrine cause of hypercalcemia is leading to parathyroid suppression (i.e., non-PTH-mediated hypercalcemia). A chest x-ray reveals bilateral hilar adenopathy and interstitial infiltrates, which are pathognomonic for sarcoidosis. Sarcoidosis is a granulomatous disorder that is classically associated with calcium homeostasis dysregulation due to the overactivation of macrophages and increased extra-renal production of 1-alpha hydroxylase. This increased enzymatic expression leads to increased conversion of 25-vit D to 1,25-vit D (i.e., calcitriol), causing increased gastrointestinal calcium and phosphate absorption, increased renal calcium reabsorption, and increased bone resorption. This is seen as hyperphosphatemia, hypercalcemia, hypervitaminosis D, and suppression of PTH levels.

      Incorrect Answers: (B, C, D, E) For more information, check out Med School Bootcamp’s videos on Sarcoidosis.
    35. Correct Answer: (A) This patient presents with vaginal pain, pruritus, and dyspareunia. Physical exam reveals edematous, erythematous vaginal mucosa with white discharge, suggestive of a Candida albicans infection. A vaginal smear reveals the presence of normal vaginal epithelium and pseudohyphae, confirming the diagnosis of vaginal candidiasis. This patient is at risk for the development of this condition based on her history of type 2 diabetes mellitus.

      Incorrect Answers: (B) In women, Chlamydia trachomatis infections commonly manifest with symptoms of abdominal or pelvic pain, dysuria, and mucopurulent cervicitis. Nucleic acid amplification testing (NAAT) is the gold standard for diagnosis. (C) Herpes simplex virus infections typically manifest with painful genital vesicles and ulcers. White cervical discharge and visualization of pseudohyphae would not be expected. (D) Infections caused by the human papillomavirus (HPV) often manifest as flesh-colored, cauliflower-like growths around the anus or on the vulva, vagina, or cervix. (E) Trichomonas vaginalis infections are associated with yellow-green, frothy, and malodorous vaginal discharge and visualization of motile, pear-shaped organisms (i.e., trichomonads) on wet prep microscopy.
    36. Correct Answer: (C) This patient's presentation is consistent with a spinal cord injury; the pattern of weakness, sensory loss, and reflexes suggests a lesion at the T1 spinal cord level (i.e., at the level of the clavicle) or below. Patients with a T1 spinal cord injury exhibit motor deficits of the lower extremities and trunk, while the upper extremities remain unaffected. Sensory loss occurs below the T1 level. Hyperreflexia, especially in the lower limbs, indicates disrupted descending inhibitory pathways. Muscle atrophy may develop over time in the lower limbs and trunk due to disuse and neural loss.

      Incorrect Answers: (A & B) A spinal lesion located at either C5 or C7 would cause deficits in the upper extremity, which was not observed in this patient. The presence of normal deep tendon reflexes in the biceps and triceps indicates normal functioning in these nerves. (D & E) A lesion occurring at T3 would reveal sensation loss beginning at the upper part of the chest and sternum; in a lesion at T5, sensation loss would begin at the upper abdomen and the lower chest region. This patient’s physical examination showed decreased pinprick sensation at the level of the clavicle. This sensation would be intact if the lesions occurred in either the T3 or T5 cord levels.
    37. Correct Answer: (D) This patient has evidence of autosomal dominant familial hypercholesterolemia, supported by tendinous xanthomas on the hands and Achilles, familial history of early onset atherosclerosis (indicating coronary artery disease), and hypercholesterolemia on serum lipid studies. Familial hypercholesterolemia is characterized by LDL-receptor deficiency, leading to impaired clearance of LDL cholesterol from the bloodstream and subsequent LDL accumulation, foam cell formation, and development of atherosclerotic plaques in arteries throughout the body.

      Incorrect Answers: (A) Abetalipoproteinemia is caused by Apo B48 deficiency and manifests with steatorrhea, failure to thrive, and deficiency in fat-soluble vitamins. Laboratory evaluation would reveal the absence of chylomicrons, VLDL, and LDL. (B & E) Patients with Apo C2 or lipoprotein lipase deficiency commonly develop familial hyperchylomicronemia with hypertriglyceridemia and its possible sequelae (e.g., pancreatitis, eruptive xanthomas) with no increased risk of atherosclerosis. (C) Symptoms of HMG-CoA reductase deficiency include vomiting, hypotonia, weakness, metabolic acidosis without ketonuria, and hypoglycemia on laboratory evaluation; they often occur within the first year of life.
    38. Correct Answer: (E) This patient's presentation of severe, sharp upper back pain, hypotension, tachycardia, and a late diastolic murmur at the lower left sternal border is highly suggestive of a stretched aortic annulus (i.e., aortic root dilation). This finding often occurs in individuals with connective tissue disorders (e.g., Marfan syndrome). The late diastolic murmur is characteristic of aortic regurgitation (i.e., aortic insufficiency), which can result from the dilation of the aortic annulus and lead to incomplete closure of the aortic valve cusps.

      Incorrect Answers: (A) Physical evaluation of a patient with an atrial septal defect would reveal a mid-systolic ejection murmur heard at the upper left sternal border. (B) Mitral stenosis is associated with a low-pitched diastolic rumbling murmur with an opening snap that is best heard at the apex. (C) Papillary muscle ruptures most often occur secondary to myocardial infarction and are associated with a high-pitched holosystolic murmur with a blowing quality best heard at the apex. (D) A perforated tricuspid valve may occur in IV drug users secondary to endocarditis or as a result of improper cardiac pacemaker placements. Auscultation will reveal a pansystolic murmur with a blowing quality typically heard along the lower left sternal border. For more information, check out Med School Bootcamp’s videos on Valvular Disease.
    39. Correct Answer: (B) This patient's presentation of double vision with the inability of the left eye to adduct past the midline on horizontal gaze when looking to the right suggests internuclear ophthalmoplegia (INO). INO is typically caused by a lesion in the medial longitudinal fasciculus (MLF), specifically on the side opposite to the affected eye's limitation in adduction. In this case, the left eye is unable to adduct when looking to the right, indicating damage to the left MLF.

      Incorrect Answers: (A & C) The left abducens nerve and the left nucleus of the abducens nerve are not responsible for this specific pattern of ocular movement impairment. (D & F) The right abducens nerve and the right nucleus of the abducens nerve are on the same side as the affected eye, so they are not the primary cause of the deficit. (E) The right MLF is not directly involved in this presentation of INO, as the primary lesion is on the left side of the brainstem.
    40. Correct Answer: (D) Type II pneumocytes (i.e., type II alveolar cells) are the cell type most likely to proliferate and reestablish the injured epithelial layer in the alveoli of a patient recovering from acute respiratory distress syndrome (ARDS). Type II pneumocytes are known for their ability to regenerate and repair the alveolar epithelium. They have stem cell-like properties and can differentiate into type I pneumocytes, which are responsible for gas exchange and play a crucial role in surfactant production to maintain alveolar stability. During ARDS, the alveolar epithelium is often damaged, leading to impaired gas exchange and lung function. The proliferation and differentiation of type II pneumocytes help in the repair and regeneration of the damaged alveolar epithelial layer, contributing to the recovery of lung function in patients with ARDS.

      Incorrect Answers: (A) This is an endothelial cell that is responsible for lining blood vessels and facilitating the exchange of oxygen and carbon dioxide between the bloodstream and the alveoli. (B) This is a red blood cell. (C) This is a type I pneumocyte. These cells comprise the majority of the alveolar epithelium, and their main function is to facilitate gas exchange. (E) This is an alveolar macrophage; it is responsible for the phagocytosis of inhaled pathogens.

    Block 2 

    1. Correct Answer: (B) This 11-year-old female has scant pubic hair, the initial formation of a breast bud (i.e., areolar tissue enlargement), and an increase in areolar diameter. These findings are characteristic of breast and pubic hair development in Tanner stage 2 and indicate the beginning of puberty. Thelarche (i.e., breast development) classically precedes a growth spurt and pubarche (i.e., pubic hair growth) in females.

      Incorrect Answers: (A, C, D, E) For additional information, check out Med School Bootcamp's video on Childhood Development.
    2. Correct Answer: (A) This young male had an episode of bloody stool consisting of hard “pebbles” and blood on the toilet tissue when wiping himself. His most recent bowel movement was 5 days prior and caused him pain. Further examination reveals abdominal distension, hypoactive bowel sounds, and a rectal mucosal prolapse. These findings are most likely due to constipation, which can lead to abdominal pain and a need for increased intraabdominal pressure during defecation, leading to anal fissures, hemorrhoids, and/or rectal prolapse.

      Incorrect Answers: (B) Gastrointestinal symptoms in cystic fibrosis often include failure to thrive and steatorrhea due to improper absorption of fat-soluble vitamins. The absence of pulmonary symptoms makes this choice unlikely. (C) Hirschsprung disease often manifests in infants as abdominal distention, bilious emesis, and failure to pass meconium within 48 hours. (D) Whipworm infestation (Trichuris trichiura), not hookworm infection, is a potential cause of rectal prolapse. This is a very rare occurrence in developed countries. (E) Intussusception often manifests in young children as intermittent, severe, abdominal pain and “currant jelly” dark red stools. Pain may be lessened when the patient brings their legs to the chest. For additional information, check out Med School Bootcamp's video on Veins, Anastomoses, and Hemorrhoids.
    3. Correct Answer: (C) This patient presents with fever, chills, severe headache, weakness, muscle pain, vomiting, diarrhea, and abdominal pain, which are indicative of a viral infection. The patient recently returned from a medical mission trip to West Africa, which is a region known for viral infections such as Ebola and Lassa fever. The reverse transcription-polymerase chain reaction (RT-PCR) test is the most sensitive and specific test for detecting viral RNA or DNA in a patient sample; it can identify the presence of the viral genome even in low concentrations.

      Incorrect Answers: (A, B, D, E) For information about these methods, check out Med School Bootcamp’s videos on Laboratory Techniques.
    4. Correct Answer: (C) The structures labeled by the X’s most likely depict unmyelinated axons of the peripheral nervous system (PNS). These axons include C fiber nociceptors, postganglionic sympathetic fibers, and some preganglionic sympathetic and parasympathetic fibers. In normal cutaneous tissue, unmyelinated C fibers play a crucial role in transmitting slow, dull, and poorly localized sensations, which are characteristic of pain and temperature. When these unmyelinated axons are damaged or lost (i.e., small fiber neuropathy), the patient's ability to perceive pain and temperature can be impaired.

      Incorrect Answers: (A, B, D, E) For information about nervous system histology, check out Med School Bootcamp’s videos on the Nervous System.
    5. Correct Answer: (C) This infant's coughing spells that are often followed by vomiting are suggestive of pertussis (i.e., whooping cough), which is caused by Bordetella pertussis. Macrolide antibiotics (e.g., azithromycin, erythromycin) are the treatment of choice for pertussis in both infants and older children. They are effective in reducing the severity and duration of symptoms and preventing further spread of the disease.

      Incorrect Answers: (A, B, D, E) Cephalosporins, fluoroquinolones, penicillins, and sulfonamides may be considered as alternative treatment options for whooping cough infections if macrolide antibiotics are contraindicated; since this is not the case in the patient, a macrolide antibiotic would be the most appropriate first-line treatment. For more information, check out Med School Bootcamp’s videos on Antibiotics.
    6. Correct Answer: (B) Desensitization therapy involves gradually exposing a patient to increasing doses of an allergen (e.g., cat allergen). Over time, this exposure leads to the production of IgG antibodies specific to the allergen that compete with IgE antibodies for binding to the allergen. As a result, the IgG antibodies block allergen binding to mast cells, preventing the release of histamine and other mediators responsible for allergic symptoms. This mechanism helps reduce the allergic response and provides relief from symptoms.

      Incorrect Answers: (A, C, D, E) These answer choices do not describe the mechanism associated with allergen desensitization therapy.
    7. Correct Answer: (A) This 3-week-old female presents with one week of jaundice, scleral icterus, acholic stools, and dark urine. Other remarkable findings include hepatomegaly, direct hyperbilirubinemia, and mild elevations in AST/ALT. This clinical picture in a newborn is consistent with biliary atresia, in which an infant is capable of conjugating bilirubin but is unable to secrete it into the small intestine. This leads to increased total and direct bilirubin and subsequent hepatomegaly, varying levels of transaminitis, and symptoms of hyperbilirubinemia (e.g., jaundice, scleral icterus).

      Incorrect Answers: (B) Crigler-Najar syndrome type I is characterized by increased indirect bilirubin due to the inability to conjugate. (C) Gilbert syndrome is a benign condition characterized by a decreased ability to conjugate bilirubin that typically manifests in adolescence as benign jaundice in periods of physiologic stress (e.g., surgery, infection, fasting, alcohol use). Though similar to Crigler-Najar syndrome, Gilbert syndrome is less severe. (D) Hemolytic disease of the newborn manifests with significant intravascular hemolysis that can lead to severely increased indirect hyperbilirubinemia and possible kernicterus. (E) Physiologic jaundice is an indirect hyperbilirubinemia that manifests after the first day of life. It is a benign condition that occurs due to a decreased life of fetal erythrocytes and immature hepatic conjugation of bilirubin. For additional information, check out Med School Bootcamp's video on Jaundice.
    8. Correct Answer: (A) This patient presents with medically uncontrolled hypertension. Lab studies reveal increased renin in the absence of other significant findings. Angiography reveals a high-grade stenotic lesion of the proximal right renal artery, which is pathognomonic for atherosclerotic renal artery stenosis. This leads to relative hypoperfusion of the right kidney, which causes increased activation of the renin-angiotensin-aldosterone system in the right kidney and ultimately leads to hypertension that is difficult to control. The unaffected kidney typically maintains adequate electrolyte balance, explaining the normal electrolytes in this patient.

      Incorrect Answers: (B) Congenital renal artery hypoplasia is a condition in which one or both of the renal arteries are underdeveloped or smaller than average from birth. Since this patient’s elevated blood pressure readings were first discovered three weeks ago, a congenital cause is unlikely. (C) Fibromuscular dysplasia is a common cause of distal renal artery stenosis that classically affects young women. (D) Takayasu arteritis is an autoimmune large vessel vasculitis of the aortic arch that typically occurs in Asian females under the age of 40. Symptoms often include weakened upper extremity pulses, fever, and night sweats. (E) Temporal arteritis is a large-cell vasculitis that commonly occurs in women over the age of 50; it is associated with unilateral headache and jaw claudication. For additional information, check out Med School Bootcamp's video on Secondary Hypertension.
    9. Correct Answer: (B) This patient presents with anemia as well as advanced kidney disease as a result of prolonged use of analgesics. The most likely cause of her anemia is decreased erythropoietin production. The kidneys are the primary source of erythropoietin, a hormone that stimulates red blood cell production. Erythropoietin production is diminished in kidney disease, leading to anemia. This patient's laboratory findings (e.g., low hemoglobin and hematocrit levels), elevated ferritin (likely due to inflammation from chronic disease), and positive occult blood in the stool (indicating gastrointestinal bleeding) all point towards anemia related to decreased erythropoietin production.

      Incorrect Answers: (A) Bone marrow suppression most often occurs secondary to immunosuppressive therapy or inherited genetic conditions (e.g., Fanconi anemia). Laboratory evaluation would reveal decreased levels of all hematopoietic cell types (e.g., RBC, WBC, platelets). (C) Intravascular hemolysis is associated with increased ferritin, total iron, transferrin saturation, and lactate dehydrogenase levels due to their release from hemoglobin; mean corpuscular volume (MCV) tends to be normal or increased. (D) Iron deficiency anemia is associated with decreased MCV, ferritin, total iron, and transferrin saturation due to insufficient iron levels in the body. (E) Splenic sequestration most commonly occurs in the setting of sickle cell anemia; it manifests with the abrupt onset of pallor, weakness, and tachycardia and a decrease in hemoglobin levels due to blood trapping in the spleen.
    10. Correct Answer: (D) This patient has developed a urinary tract infection (UTI), as evidenced by dysuria, increased urinary frequency, microscopic hematuria, pyuria, and nitrite/leukocyte esterase/bacteria positivity. This is likely due to a gram-negative bacterium such as Escherichia coli. Risk factors for the development of UTIs include female sex, sexual intercourse, recent history of a UTI, abnormalities of the urinary tract (e.g., BPH, nephrolithiasis, congenital malformations), pregnancy, prior conditions (e.g., diabetes mellitus, immunosuppression), and prolonged indwelling urinary catheter usage (CAUTI). UTIs are more common in females due to the shorter length of the urethra and the close proximity of the anal-genital regions. This combination allows easier bacterial spread from the anal region, vaginal colonization, and subsequent ascension of the urinary tract.

      Incorrect Answers: (A) Leiomyomata uteri (i.e., uterine fibroids) often manifest with an enlarged, irregularly shaped uterus, heavy menstrual bleeding, and pelvic pain. (B & C) Although this patient’s history of irregular periods could be consistent with perimenopause or pregnancy, it does not explain this patient’s dysuria and the presence of bacteria on urinalysis.
    11. Correct Answer: (D) This patient’s presentation of itchy patches of skin and a burning sensation in the feet, along with the physical exam findings shown in the photograph, is indicative of a dermatophyte fungal infection, specifically tinea pedis (i.e., athlete's foot). Tinea pedis is caused by dermatophyte fungi that invade the stratum corneum of the skin. Squalene monooxygenase is an enzyme involved in the synthesis of ergosterol, an essential component of fungal cell membranes. Inhibition of squalene monooxygenase disrupts ergosterol synthesis, leading to the weakening and disruption of fungal cell membranes. This ultimately inhibits fungal growth and is the mechanism of action of drugs like terbinafine, which are commonly used to treat tinea pedis and other dermatophyte infections.

      Incorrect Answers: (A, B, C, E) Inhibition of these enzymes does not describe the mechanism of the first-line treatment for tinea pedis. For additional information, check out Med School Bootcamp's videos on Tinea.
    12. Correct Answer: (B) This patient has developed signs of meningismus (e.g., fever, headache, nuchal rigidity, photophobia) after similar symptoms in his close contacts. Lumbar puncture findings (e.g., increased opening pressure, lymphocytosis, increased protein, normal glucose) are consistent with a viral cause. Enteroviruses (e.g., echoviruses, coxsackieviruses) are the most common cause of viral meningitis in all age groups.

      Incorrect Answers: (A & C) Even though these viruses could result in the cerebrospinal fluid findings observed in this patient, enterovirus is the most common cause of viral meningitis, making it the more likely culprit. (D & E) Meningitis caused by bacterial sources (e.g., Neisseria meningitidis, Streptococcus pneumoniae) is associated with decreased glucose, elevated protein, and neutrophils in cerebrospinal fluid. For more information, check out Med School Bootcamp’s video on Infectious Neuropathology.
    13. Correct Answer: (A) The presence of a calcium deposit over the greater tubercle is indicative of calcific tendinitis of the shoulder. This condition can cause pain and limited range of motion. In this case, the calcium deposit is located near the attachment of the supraspinatus muscle, which mediates the first 15° of shoulder abduction from a neutral position. Therefore, this patient is likely experiencing reduced ability to abduct the shoulder.

      Incorrect Answers: (B, C, D, E) For more information, check out Med School Bootcamp’s video on Shoulder Anatomy.
    14. Correct Answer: (B) Being honest about one’s limited experience in managing a medical condition can build trust with the patient, as it shows honesty and a commitment to transparency. This physician’s willingness to learn more about chronic fatigue syndrome indicates that they are committed to understanding the patient's perspective and working toward a better outcome. This is an example of a positive patient-provider relationship that can enhance the patient’s overall quality of care.

      Incorrect Answers: (A, C, E) It would not be appropriate to respond in a way that might imply that the patient is making up her experience. (D) It would not be appropriate to ignore the patient’s past medical records and history. For additional information, check out Med School Bootcamp's video on Disorders of Somatization and Hypochondriasis.
    15. Correct Answer: (E) This patient is experiencing occupational-related allergic contact dermatitis due to latex exposure. This is characterized by a type IV hypersensitivity reaction, which causes delayed symptoms following exposure to latex (e.g., gloves). Contact dermatitis is largely a clinical diagnosis and is evidenced by a limited erythematous rash that is confined to the area of exposure. Other causes include exposure to poison ivy/oak/sumac, nickel, neomycin, perfumes, soaps, and many others. Latex is a very common cause of allergic contact dermatitis.

      Incorrect Answers: (A, B, C, D) The distribution of this patient’s rash (i.e., only involving the skin that directly contacts his gloves) lowers the likelihood that his rash is due to these answer choices. For additional information, check out Med School Bootcamp's video on Type IV Hypersensitivity Reactions.
    16. Correct Answer: (D) This response allows the physician to engage with the patient and her concerns in a supportive and collaborative manner. It indicates a willingness to have a thorough discussion about her birth plan, including the potential risks and benefits of a home birth. It also provides an opportunity for the physician to educate the patient about the safety and medical considerations surrounding childbirth and explore her reasons for choosing a home birth. This approach is more likely to lead to constructive dialogue and shared decision-making regarding the patient's birth plan while ensuring that the patient's health and well-being remain a priority.

      Incorrect Answers: (A, B, C, E) Before making decisions that could negatively impact the physician-patient relationship (i.e., refusing to provide care, speaking with others about the patient’s care, referring care to the midwife, etc.), the physician should obtain as much information as possible about the patient’s desires for her birth plan.
    17. Correct Answer: (C) This patient is unknowingly pregnant while taking an ACE inhibitor (i.e., lisinopril). This is a teratogenic agent known to cause cardiovascular and CNS malformations during the first trimester and fetal renal damage with resulting oligohydramnios, impaired cranial ossification, and potential fetal demise during the second and third trimesters. ACE inhibitors are thought to produce these effects via inhibition of fetal urine production due to impaired renal blood and filtration flow dynamics.

      Incorrect Answers: (A, B, D, E) Nothing in the mother's medical history suggests that these processes would be subject to interference. For additional information, check out Med School Bootcamp's video on Teratogenic Medications.
    18. Correct Answer: (E) This patient presents with worry about memory loss. Though he has some signs of forgetfulness, he lives alone and successfully manages his activities of daily living (e.g., feeding, toileting, dressing) and instrumental activities of daily living (e.g., finances, cooking, shopping), which indicates he is able to live independently and continue working part-time. His exam is unremarkable, and he is found to have a full range of affect, euthymic mood, intact concentration, math skills, proper object recognition and recall, and command recognition and enaction. The most appropriate response is to counsel the patient that some forgetfulness is normal with aging. The physician can recommend methods to help mitigate age-related memory decline, such as regular physical exercise, maintaining a healthy diet, and adequate sleep hygiene.

      Incorrect Answers: (A & C) There is no evidence of pathological memory loss in this patient’s history and exam. (B & D) Continuing this patient’s workup without suspicion of an underlying pathology would be inappropriate.
    19. Correct Answer: (E) This 42-year-old female with a previous history of two benign breast cysts now presents with a new cyst that is increasing in size and tenderness. Mammography shows a 3.1 cm irregular mass in the left breast, and photomicrographs of the excisional biopsy specimen show pleomorphic irregular glands. Pleomorphism refers to variations in the size and shape of cells and their nuclei, and irregular glands suggest architectural disorganization. These findings are not a characteristic feature of benign breast lesions but are concerning for malignancy, particularly breast carcinoma.

      Incorrect Answers: (A, B, C, D) These findings on histopathological evaluation are consistent with benign fibrocystic changes of the breast, which is a common condition that occurs most often in women between the ages of 30 and 50 years. It is characterized by localized proliferation of the breast ducts, lobules, and stroma.
    20. Correct Answer: (E) This patient has developed myalgia, fatigue, cough, sore throat, rhinorrhea, fever, tachycardia, tachypnea, hypertension, and a severe, painful, rapidly spreading bullous rash over her body and mouth that began 10 days after treatment with a sulfonamide antibiotic (i.e., TMP-SMX). Physical exam reveals a positive Nikolsky sign, and biopsy reveals diffuse epidermal keratinocyte necrosis with minimal lymphocytic infiltration in the dermis. This is a classic description of Stevens-Johnson syndrome (SJS) and toxic epidermal necrolysis (TEN). TEN involves > 30% of the body surface area, while SJS involves < 10%. The most common cause of SJS and TEN is drug exposure (e.g., sulfonamides, aminopenicillins, antiepileptics, NSAIDs, and allopurinol). This patient’s rash is likely caused by a delayed (i.e., type IV) hypersensitivity reaction of drug-specific cytotoxic T lymphocytes, leading to the release of cytolytic proteins (e.g., granulysin) and subsequent keratinocyte damage. Other causes include infection (e.g., CMV, HSV, mycoplasma), vaccinations, graft-versus-host disease (GVHD), and various unknown mechanisms.

      Incorrect Answers: (A) Erythema multiforme is a hypersensitivity reaction that often manifests with multiple targetoid lesions; it is associated with recent herpes simplex infections or drug exposure. (B) Linear IgA bullous dermatitis occurs due to circulating IgA anti-basement membrane zone antibodies. It typically causes tense blisters arranged in a “string-of-pearls” configuration on an erythematous base. In children, the anogenital area and lower abdomen are most often affected, while the trunk and extensor surface of the limbs are more likely involved in adults. (C) Pemphigus vulgaris is an autoimmune condition that is caused by IgG antibodies against desmoglein-1 in the epidermis. It most often occurs in older adults and manifests with flaccid bullae, mucosal involvement, and a positive Nikolsky sign. (D) Staphylococcal scalded skin syndrome (SSSS) is most commonly seen in newborns and is characterized by fever and generalized erythematous rash with a positive Nikolsky sign due to the release of exotoxin. SSSS involves destruction only within the epidermis, while TEN involves both the epidermis and dermis. For additional information, check out Med School Bootcamp's video on Reactive Disease.
    21. Correct Answer: (D) In healthcare settings, a professional interpreter must be used whenever a patient is not fluent in English. It would not be appropriate to delay care or communications with the patient and their family to wait for the hospital interpreter, so using a telephone interpreter service is the most appropriate option in this scenario.

      Incorrect Answers: (A, B, C, E) For more information, check out Med School Bootcamp’s videos on Communication
    22. Correct Answer: (A) The correlation coefficient is a value between -1 and 1 that describes the strength of a linear relationship between two variables. A correlation coefficient less than 0 means that the slope of the linear relationship is negative. A correlation coefficient greater than 0 means that the slope of the linear relationship is positive. A correlation coefficient of 0 means there is no linear relationship. In this scenario, the correlation of 0.6 means that the two variables (i.e., serum LDL and serum hs-CRP) have a positive linear relationship, indicating that high values of serum LDL will be associated with high values of hs-CRP.

      Incorrect Answers: (B) A correlation coefficient of 0.6 suggests a positive, not negative, correlation between LDL and hs-CRP. (C & D) It is important to note that an association between two variables does not imply that one causes the other.
    23. Correct Answer: (C) This patient has a 10-year history of heartburn that occurs after known triggers. His condition has been uncomplicated (e.g., no weight loss, dysphagia, stricture). His physical exam is unremarkable, but endoscopy reveals erythema and ulceration of the distal esophagus above the Z line. This represents gastroesophageal reflux disease (GERD). GERD most commonly occurs due to transient loss of lower esophageal tone and reflux of gastric contents into the distal esophagus.

      Incorrect Answers: (A, B, D, E) This patient has no indication of decreased saliva production (no evidence of smoking), esophageal dysmotility (no dysphagia), or gastrinoma (no duodenal ulcer). For additional information, check out Med School Bootcamp's video on Esophagitis.
    24. Correct Answer: (B) This patient presents with key findings of plethora and splenomegaly in the setting of normal oxygen saturation, increased hemoglobin, hematocrit, leukocyte count, and leukocyte alkaline phosphatase level with occasional giant platelets observed on peripheral blood smear. These findings are consistent with polycythemia vera (PV), a malignant hematopoietic neoplasm caused by abnormally high activity of transduction of growth signals in the kinase domain. JAK2 is integral to the regulation of erythropoiesis, thrombopoiesis, and granulopoiesis. The primary hematologic defect in PV is usually a JAK2 gain-of-function mutation at the level of the hematopoietic stem cells, meaning that it can affect multiple cell lines. Note that an abnormally low oxygen saturation would be more suggestive of a secondary polycythemia, as observed in COPD.

      Incorrect Answers: (A, C, D, E) Defects in the production of these cell types would not lead to the development of PV. For additional information, check out Med School Bootcamp's video on Polycythemia Vera.
    25. Correct Answer: (A) This patient presents with a mild injury and is subsequently found to have a megaloblastic macrocytic anemia, as evidenced by a hypersegmented neutrophil (> 5 lobes) and erythrocytes with central pallor. In the context of significant daily alcohol intake (i.e., 4-6 beers/day and double on weekends) and rudimentary dietary intake (i.e., “tea and toast” diet), the most likely cause of this patient’s anemia is folic acid (i.e., folate) deficiency. Affected patients are unable to maintain the 1-carbon pool due to their inability to regenerate folate equivalents (e.g., tetrahydrofolate). This impairs the conversion of homocysteine to methionine, leading to hyperhomocysteinemia and potential cardiovascular sequelae. Lack of folate cycling also prevents the conversion of dUMP to dTMP for DNA synthesis.

      Incorrect Answers: (B, C, D, E) Deficiencies in these substances would not explain the findings observed in this patient. For additional information, check out Med School Bootcamp's video on Folate Deficiency.
    26. Correct Answer: (C) This otherwise healthy newborn has a penile-appearing structure with hypospadias located above the vaginal introitus. If this infant is found to have a 46, XX karyotype, it is most likely caused by maternal exposure to androgens while pregnant, which results in virilization.

      Incorrect Answers: (A) 17alpha-hydroxyprogesterone is converted to 11-deoxycortisol and then cortisol. Deficiency would lead to adrenal insufficiency, hypertension, and impaired stress response but not virilization, as less 17 alpha-hydroxyprogesterone would be converted to androstenedione for testosterone production. (B & D) Increased concentration of Mullerian-inhibiting substance and/or the presence of sex-determining region Y would lead to the expression of Wolffian duct structures and an absence of the uterus, fallopian tubes, and upper vagina. Ambiguous external genitalia would be unlikely in this case because the XX fetus would produce estrogen, resulting in the formation of female external genitalia. (E) 5alpha-reductase deficiency is a disorder of 46, XY males in which testosterone cannot be converted to dihydrotestosterone (DHT). This prevents the adequate maturation of the external male genitalia during development, resulting in internal male genitalia and external female-appearing genitalia. This classically resolves at puberty as the substantial increase in testosterone leads to the maturation of male external genitalia. For additional information, check out Med School Bootcamp's video on Disorders of Sexual Development.
    27. Correct Answer: (C) This patient, who lives with family members and has a complex medical history, has many risk factors for falls, including bilateral below-the-knee amputations with wheelchair dependence, elderly age, Alzheimer’s dementia, medications (e.g., amlodipine, insulin), and alcohol intake. She is found to have scattered bruising over her abdomen (likely due to insulin administration) and all extremities (perhaps due to falls). Though this patient has a potential mechanism and medical explanation for her physical condition, it is important to assess her relationship with her family to ensure she is receiving adequate care while also ruling out elderly abuse. Medical care providers are mandatory reporters of elderly abuse and must be aware of potential warning signs, including unexplained injuries, changes in behavior or mood, social withdrawal, financial exploitation, and neglect of medical needs or personal hygiene.

      Incorrect Answers: (A, B, D, E) Before obtaining any additional histories, the physician must first assess the patient’s safety and the risk of elder abuse.
    28. Correct Answer: (C) Red-green color blindness follows a pattern of X-linked recessive inheritance. Since males have one X chromosome and one Y chromosome (XY), they only inherit one copy of the X-linked gene. If this X chromosome carries the recessive allele, males will express the trait or disorder associated with it because they lack a second X chromosome with a normal allele to mask the effects. In contrast, females have two X chromosomes (XX). If they inherit one X chromosome with the recessive allele and one with a normal allele, they are considered carriers. To express the trait or disorder, females would need to inherit two copies of the recessive allele (i.e., one from each parent), making it less common for females to be affected.

      In this pedigree, the father (II,2) has red-green color blindness, which means he carries one X chromosome with the gene for red-green color vision deficiency (Xc) and one Y chromosome. Similarly, the maternal grandfather (I,2) has red-green color blindness. This indicates that the mother (II,1) carries one X chromosome with the Xc gene and one X chromosome with normal color vision (X). If this couple were to have a son, he would inherit one X chromosome from the mother, which has a 50% chance of being the Xc gene, and one Y chromosome from the father. Therefore, there is a 50% chance that the son will inherit the X chromosome with the Xc gene from the mother and will be affected by red-green color blindness as a result.

      Incorrect Answers: (A, B, D, E) For more information about inheritance patterns, check out Med School Bootcamp’s videos on the Basics of Genetic Inheritance.
    29. Correct Answer: (C) This patient is experiencing dysmenorrhea without other significant medical issues. The first-line treatment in an otherwise healthy young female is non-steroidal anti-inflammatory drugs (NSAIDs), such as ibuprofen. Primary dysmenorrhea occurs due to endometrial prostaglandin production that leads to localized vasoconstriction and ischemia as well as increased uterine contractions in order to prevent blood loss. This is experienced by patients as intense lower abdominal cramping and discomfort that typically radiates to the back. The use of NSAIDs prevents prostaglandin synthesis, improving these symptoms. Other methods of relief include local heat application (e.g., heating pad) and implementation of oral contraception (OCPs).

      Incorrect Answers: (A, B, D, E) For additional information, check out Med School Bootcamp's video on Menstrual Cycle Disorders.
    30. Correct Answer: (D) This patient has developed left-sided hydronephrosis in the setting of recurrent urinary tract infections. The T2-weighted MRI shows a proximal obstruction at the level of the ureteropelvic junction (i.e., where the ureter leaves the renal collecting pelvis). Hydronephrosis develops as a result of increased back pressure on the proximal renal system. Obstructed areas are viewed as dark areas on a T2-weighted MRI surrounded by areas of relatively diminished intensity due to some contrast leaking past the obstruction.

      Incorrect Answers: (A, B, C) These are uncommon sites of obstruction and are unlikely to lead to this patient’s symptoms. (E) While the ureterovesical junction (i.e., the site where the ureter enters the bladder) is a relatively common site of obstruction, the most common site of obstruction is the ureteropelvic junction, making it the best answer.
    31. Correct Answer: (A) The formula for absolute risk reduction (ARR) is calculated as the risk in the riskier procedure minus the risk in the less risky procedure. The question states that the risk for wound infection for Procedure A is 12/1000, and the risk for Procedure B is 18/1000. Therefore, ARR = Procedure B – Procedure A = (18/1000) – (12/1000).

      Incorrect Answers: (B, C, D, E) For additional information, check out Med School Bootcamp's video on Additional Calculations of Relative Risk.
    32. Correct Answer: (B) This patient with cirrhosis (i.e., irregular liver surface, jaundice, caput medusae) has developed esophageal variceal bleeding and is about to undergo a transjugular intrahepatic portosystemic shunt (TIPS) procedure to decrease portal venous pressure and relieve pressure within anatomically relevant portocaval anastomoses (e.g., left gastric: esophageal, paraumbilical: small gastric, superior rectal: middle/inferior rectal). During this procedure, a needle catheter is inserted into the internal jugular vein and passed to the hepatic vein. Once inside, the needle pierces the liver parenchyma, creating a new shunt between the portal vein and the inferior vena cava.

      Incorrect Answers: (A, C, D, E) These choices are all veins within the portal venous system. Creating a shunt between one of these and another portal venous structure would not relieve intraportal pressure. For additional information, check out Med School Bootcamp's video on Veins, Anastomoses, and Hemorrhoids.
    33. Correct Answer: (A) This statement encourages the patient to explore their motivations for changing behavior. Motivational interviewing involves a patient-centered, collaborative approach to strengthen the patient's commitment to behavior change through reflective questioning and promotion of intrinsic motivation.

      Incorrect Answers: (B, C, D, E) Conversational tactics that invoke fear or shaming are often not effective in promoting behavioral change and can weaken the physician-patient relationship. For additional information check out Med School Bootcamp's video on the Transtheoretical Model of Change.
    34. Correct Answer: (A) This patient is presenting with bilateral parotid and submandibular gland swelling, mouth pain, discoloration of teeth, and scarring on the dorsum of the hand. These are classic signs and symptoms of bulimia nervosa, which is characterized by recurrent episodes of binge eating followed by inappropriate compensatory behaviors such as self-induced vomiting or laxative use. Anticholinergic medications (e.g., scopolamine or atropine) bind to muscarinic acetylcholine receptors to help alleviate the parasympathetic symptoms associated with bulimia nervosa, particularly the excessive salivation and gastrointestinal symptoms that result from repeated self-induced vomiting.

      Incorrect Answers: (B, C, D, E) For additional information, check out Med School Bootcamp's video on Eating Disorders.
    35. Correct Answer: (E) This patient at 15 weeks gestation presents to the hospital in active labor. The medical team has assessed the situation and determined that the baby's condition is not compatible with life outside the womb. In such cases, it is the physician's ethical and medical responsibility to provide the patient with information about the situation in a clear and compassionate way.

      Incorrect Answers: (A, B, C, D) While empathy is essential in delivering such news, it is crucial to provide the patient with honest and realistic information about the prognosis. This allows the patient to make informed decisions regarding their care and the care of the baby, even if the outcome is extremely distressing.
    36. Correct Answer: (C) This patient’s presentation of nonhealing, painless ulcers on the neck and arms, along with a history of sleeping in a region with a warm climate, is indicative of cutaneous leishmaniasis. Cutaneous leishmaniasis is caused by protozoa of the Leishmania species and is transmitted to humans through the bite of infected sandflies.

      Incorrect Answers: (A) Fleas are most commonly involved in the spread of infections such as the plague (caused by Yersinia pestis) and endemic typhus (caused by Rickettsia typhi). (B) Mosquitos are vectors for several illnesses (e.g., malaria, Dengue virus, Chikungunya virus, yellow fever virus, Zika virus, and West Nile virus). None of these viruses are consistent with the constellation of symptoms observed in this patient. (D) Brown recluse and black widow spider bites are commonly tested on the USMLE exam. A brown recluse spider bite often manifests with a painless lesion that later becomes an inflammatory and necrotic ulcer, while a black widow bite is often associated with pain and a targetoid lesion with systemic symptoms such as emesis, abdominal cramps, and paralysis. (E) Ticks are the vector for several infectious conditions (e.g., Lyme disease, Rocky Mountain spotted fever, babesiosis, ehrlichiosis, and anaplasmosis). None of these infections are consistent with the constellation of symptoms observed in this patient.
    37. Correct Answer: (D) This patient has classic symptoms of hypokalemic periodic paralysis, an autosomal dominant disorder characterized by recurrent episodes of muscle weakness that typically develop after certain triggers, including strenuous exercise (e.g., football), alcohol ingestion, dietary changes, medication use, and stress. Muscular paralysis/weakness in patients with hypokalemic periodic paralysis occurs due to mutations in sodium and calcium channel genes that cause intracellular shifting of serum potassium. Laboratory analysis would most likely show hypokalemia.

      Incorrect Answers: (A) Although calcium imbalances may lead to muscle weakness, symptoms of hypocalcemia typically include muscle cramps, spasms, and tingling sensations (i.e., paresthesia). Additionally, acute ingestion of high-carbohydrate meals is not strongly associated with the development of hypocalcemia. (B) Abnormal levels of chloride are not associated with periodic paralysis. (C) Alterations in magnesium, specifically hypomagnesemia, manifest with symptoms such as muscle cramps, generalized weakness, and tremors. The development of hypomagnesemia is not strongly associated with high-carbohydrate meals or strenuous exercise. (E) Disturbances in sodium levels, such as in hyponatremia, typically demonstrate slow-onset, nonspecific manifestations, such as generalized weakness, malaise, and fatigue.
    38. Correct Answer: (C) ​​During pregnancy, maternal prolactin levels increase, leading to the development of mammary glands and secretion of colostrum in preparation for breastfeeding. Some of this maternal prolactin can cross the placenta and affect the newborn's breast tissue. This exposure to maternal prolactin can result in a temporary condition called "witch's milk," which refers to milk-like discharge from a newborn's nipples. It is a benign condition that usually resolves on its own within a few weeks.

      Incorrect Answers: (A, B, D, E) None of these hormones play a significant role in causing witch’s milk in a newborn.
    39. Correct Answer: (A) The arrow is pointing to this patient’s recently replaced aortic valve. The aortic valve is found in the second intercostal space at the right sternal border. A prosthetic click is commonly heard when a mechanical or bioprosthetic valve is placed in the aortic position.

      Incorrect Answers: (B, C, D) The arrow in this image is pointing to the aortic valve. The mitral valve is found in the fifth intercostal space at the left midclavicular line, the pulmonic valve is found in the second intercostal space at the left sternal border, and the tricuspid valve is found in the fourth intercostal space at the left sternal border.
    40. Correct Answer: (E) After a puncture wound to the left hand several days ago, this patient presents with diffuse swelling, erythema, and necrosis, alongside systemic symptoms (i.e., fever, shortness of breath, and palpitations). These findings are concerning for an invasive soft tissue infection. Staphylococcus aureus is a common pathogen associated with such infections, especially following puncture wounds; its ability to produce toxins can lead to severe tissue damage and systemic symptoms such as those seen in this patient.

      Incorrect Answers: (A) Clostridium tetani infections are associated with severe, diffuse muscle contractions that often occur after inoculation of bacteria into an open wound. This is unlikely in this patient because the incidence of this condition has greatly decreased following widespread vaccination efforts. (B) Soft tissue infections caused by Mycobacterium abscessus are uncommon and often manifest with painful, violaceous nodules, recurrent abscesses, or chronic sinus tracts. (C) Pasteurella multocida most commonly manifests with signs of soft tissue infection (e.g., rapidly progressing swelling, erythema, tenderness around the injury site) following the bite of a dog or a cat. (D) Pseudomonas aeruginosa infections most often occur secondary to puncture wounds in the feet or burns and manifest with a malodorous, blue-greenish coloration.

    Block 3 

    1. Correct Answer: (D) This patient with no history of trauma has developed a spontaneous vertebral fracture. She has signs of hypercortisolism (e.g., weight gain, thin lower extremities, central adiposity, purple striae), likely due to Cushing syndrome, as indicated by a high normal ACTH and elevated urinary free cortisol. The most likely cause of her fracture is an increase in osteoclast lifespan and activation (i.e., osteoclastogenesis) due to corticosteroid-mediated upregulation of RANK-L expression. This enhances the interaction between RANK-L and its receptor, (i.e., RANK) on osteoclast precursor cells, triggering signaling pathways that promote osteoclast differentiation and activity and ultimately contributing to bone loss.

      Incorrect Answers: (A, B, E, F) Calcium and phosphorus absorption and secretion are regulated primarily by parathyroid hormone and vitamin D. There is no evidence suggesting that these substances are abnormal in this patient. (C) Increased cortisol levels promote osteoclast, not osteoblast, proliferation. Paget’s disease is caused by excess osteoclastic activity followed by a compensatory increase in osteoblastic activity. This results in disorganized bone, which is less compact, mechanically weaker, and susceptible to fractures. Progression of this condition can lead to acromegaly, hearing loss, and high-output heart failure.
    2. Correct Answer: (D) This patient has developed a fever, headache, light-headedness, dizziness, chills with rigors, and myalgias after beginning therapy for Lyme disease caused by Borrelia burgdorferi. These symptoms are characteristic of a Jarisch-Herxheimer reaction, an acute, systemic reaction that occurs after initiating antibiotic therapy, most commonly with spirochete infections (e.g., Borrelia, Treponema, Leptospira). Antibiotic therapy triggers the release of toxins and cytokines (i.e., IL-6, IL-8, and TNF-α) during the destruction of bacteria, which leads to an inflammatory response including fevers, body aches, rashes, nausea, and vomiting. Symptoms typically resolve within 24 hours.

      Incorrect Answers: (A) This patient’s symptoms are a result of the release of inflammatory cytokines during the death of Borrelia burgdorferi bacteria, not an exacerbation of the infection. (B & E) Infection-mediated sepsis occurs due to the release of bacterial endotoxins that lead to widespread vasodilation and organ failure. Sepsis manifests with fever or hypothermia, tachycardia, tachypnea, altered mental status, and signs of organ dysfunction, often necessitating urgent medical intervention. (C) An IgE-mediated allergic reaction to doxycycline would manifest sooner than the Jarisch-Herxheimer reaction (i.e., minutes instead of hours) and is more likely to be associated with skin rashes (e.g., hives, angioedema) and respiratory symptoms (e.g., wheezing, shortness of breath).
    3. Correct Answer: (B) This newborn presents with ambiguous genitalia, which is suggestive of a disorder of sex development (DSD). The electrolyte and metabolic abnormalities seen in this case, including hyponatremia (i.e., low sodium) and hyperkalemia (i.e., high potassium), are indicative of salt-wasting congenital adrenal hyperplasia (CAH). CAH is commonly caused by a 21-hydroxylase deficiency, which decreases cortisol and aldosterone production and increases testosterone, thereby leading to ambiguous genitalia. Treatment is essential to manage the life-threatening consequences of salt-wasting CAH in a neonate. In this patient, the most appropriate pharmacotherapy would be exogenous aldosterone or a mineralocorticoid replacement to address the electrolyte imbalances and aid with salt reabsorption in the renal tubules.

      Incorrect Answers: (A, C, D, E) Medications targeting these receptors would not address the underlying cause of this patient’s symptoms. For more information, check out Med School Bootcamp’s video on 21-Hydroxylase Deficiency.
    4. Correct Answer: (C) This patient, who recently had Staphylococcal aureus endocarditis and is currently on intravenous oxacillin treatment, presents with fever, joint aches, and a diffuse maculopapular rash. Laboratory findings indicate acute interstitial nephritis (AIN), a hypersensitivity reaction involving the renal interstitium that can lead to acute kidney injury. AIN is often associated with drug reactions; beta-lactam antibiotics (e.g., oxacillin) are known to be potential triggers for this condition. The presence of eosinophils in the urine, along with eosinophils and WBC casts seen on urine microscopy, supports this diagnosis. AIN typically improves upon discontinuation of the offending drug and treatment with corticosteroids.

      Incorrect Answers: (A) Focal segmental glomerulosclerosis most often occurs secondary to HIV infections or heroin use and is a nephrotic syndrome characterized by proteinuria, hypoalbuminemia, and progressive renal failure. Eosinophils and WBC casts are not expected findings. (B) Glomerular hypertrophy with hemorrhage and necrosis is most suggestive of crescentic glomerulonephritis, a form of rapidly progressive glomerulonephritis (RPGN). This condition is characterized by extensive proliferation of glomerular crescents and would show RBCs and RBC casts on urinalysis due to associated hemorrhage and necrosis, respectively. (D) Diabetic nephropathy is a form of progressive kidney damage that results from mesangial expansion and glomerular basement membrane thickening. It typically manifests with albuminuria, hypertension, and eventual renal failure. A maculopapular rash with eosinophils and WBC casts are not expected findings in diabetic nephropathy. Furthermore, there is no mention of diabetes in this patient. (E) Proximal tubular dilation and loss of brush border are characteristic of acute tubular necrosis, which typically manifests with muddy-brown, granular casts, oliguria or anuria, and electrolyte disturbances (e.g., hyperkalemia and metabolic acidosis).
    5. Correct Answer: (C) This patient with a 30-year history of drinking 6-10 beers daily is seeking treatment for alcohol withdrawal. Common symptoms of alcohol withdrawal include tremors, sweating, nausea, vomiting, and, in severe cases, seizures or delirium tremens. Benzodiazepines are the first-line agents for managing alcohol withdrawal. Longer-acting agents (e.g., chlordiazepoxide, diazepam) are classically preferred in most patients, but shorter-acting agents (e.g., lorazepam, temazepam) are preferred in patients with altered drug metabolism (e.g., elderly patients or those with liver disease). Both types of agents are indirect GABA-A receptor agonists that increase the action of GABA binding by increasing the frequency of chloride channel opening. This leads to postsynaptic membrane hyperpolarization and decreased symptoms associated with neuronal excitability (e.g., anxiety, insomnia, tremors, palpitations, sweating, seizures).

      Incorrect Answers: (A) Antipsychotic medications (e.g., haloperidol) function by blocking dopamine receptors. These medications are commonly used to treat schizophrenia but are not the treatment of choice for alcohol withdrawal. (B) Dopamine reuptake inhibitors (e.g., cocaine, methylphenidate) function by decreasing the activity of dopamine transporters. These medications are not recommended treatments for alcohol withdrawal. (D) Medications that increase GABA transaminase activity would indirectly decrease GABA levels, but these medications are not often used clinically. (E) Certain drugs, such as antiepileptic medications (e.g., levetiracetam) and antipsychotic medications (e.g., clozapine), have glutamate-modulating effects, but these are not indicated for the treatment of alcohol withdrawal. (F) Serotonin reuptake inhibitors (SSRIs), such as fluoxetine and citalopram, inhibit the uptake of serotonin. These medications are used to manage many mental health conditions (e.g., major depressive disorder and generalized anxiety disorder) but are not the first-line treatment for alcohol withdrawal. (G) N-methyl-D-aspartate (NMDA) receptor agonists (e.g., ketamine) facilitate the opening of glutamate channels. (H) Serotonin receptor agonists (e.g., triptans) function by stimulating 5-hydroxytryptophan (5-HT) receptors. These are not first-line therapies for managing alcohol withdrawal. For additional information, check out Med School Bootcamp's video on Alcohol Withdrawal.
    6. Correct Answer: (A) This 6-year-old female presents with vaginal bleeding, breast buds, and minimal coarse, pigmented hair on the labia, indicating the onset of puberty. Her hormone levels show elevated testosterone, estradiol, and elevated gonadotropins (i.e., FSH and LH). These findings are likely due to central activation of neurons in the hypothalamus-pituitary-gonadal axis. Central activation of this axis leads to the release of gonadotropin-releasing hormone (GnRH), which stimulates the secretion of FSH and LH and results in the development of secondary sexual characteristics.

      Incorrect Answers: (B) Ectopic prolactin secretion is a rare condition associated with ovarian and uterine cancers, but it would not explain this patient’s presentation. Diagnosis is often made in patients who have abnormally high circulating prolactin levels on laboratory evaluation in the absence of a prolactin-producing pituitary tumor. (C) Exogenous steroid secretion would decrease testosterone, estradiol, FSH, and LH levels due to negative feedback. (D) McCune-Albright syndrome is caused by an activating mutation in the GNAS gene that results in a classic triad of polyostotic fibrous dysplasia, café-au-lait skin pigmentation, and autonomous endocrine hyperfunction (e.g., precocious puberty). (E) 21-hydroxylase deficiency typically manifests with ambiguous genitalia in females or a salt-wasting crisis in infancy.
    7. Correct Answer: (D) This previously healthy patient has developed severe left-sided flank pain, tachycardia, and hypertension, likely secondary to the pain. Further evaluation reveals microscopic hematuria and a small ureteric calculus (i.e., kidney stone) composed of uric acid. Uric acid stones are a relatively uncommon finding of nephrolithiasis, as most kidney stones are calcium-based. Patients with underlying impairments in uric acid metabolism (e.g., underexcretion or overproduction) are more likely to develop nephroliths composed of uric acid.

      When purines (e.g., adenine, guanine) are broken down, they produce uric acid that is removed from the body through filtration in the kidneys. However, if purine metabolism is increased, it can result in uric acid crystallization and stone formation. Other conditions associated with uric acid stones include gout, high cell turnover (e.g., tumor lysis syndrome, myelodysplastic syndromes), hyperuricemia, and hyperuricosuria.

      Incorrect Answers: (A, B, C) Disorders of these processes are not involved in purine metabolism and/or uric acid production. (E) Urea cycle abnormalities include ornithine transcarbamylase deficiency, arginase deficiency, carbamoyl phosphate synthetase 1 deficiency, and N-acetylglutamate synthase deficiency. These disorders involve dysfunction of ammonia conversion into urea for excretion. For additional information, check out Med School Bootcamp's video on Purine Metabolism.
    8. Correct Answer: (E) This patient with congestive heart failure is insufficiently treated with an ACE inhibitor; her physician adds a potassium-sparing diuretic (i.e., spironolactone, eplerenone, triamterene, amiloride) because of continued symptomatology. These agents work primarily within the collecting tubule of the nephron, leading to decreased sodium reabsorption and potassium excretion. This patient was most likely prescribed spironolactone, as it has been shown to reduce mortality in heart failure with reduced ejection fraction. Amiloride and triamterene are more commonly co-prescribed with thiazide diuretics to maintain a normal potassium level, while eplerenone is rarely used as an initial treatment.

      Incorrect Answers: (A, B, C, D) Structures A and B are found in the proximal convoluted tubule, and structures C and D are found in the distal convoluted tubule. Potassium-sparing diuretics exert their effects most strongly in the collecting tubule of the nephron, not in the proximal or distal convoluted tubules.
    9. Correct Answer: (C) This patient's symptoms are classic characteristics of fat malabsorption, which can lead to steatorrhea (i.e., fatty, oily stools), weight loss, and recurrent abdominal pain. Lipase is an enzyme produced by the pancreas that plays a crucial role in the digestion and absorption of dietary fats. This patient likely has impaired pancreatic lipase production, which is common in chronic pancreatitis and pancreatic insufficiency. Supplementing with lipase can improve the digestion and absorption of dietary fats, alleviate symptoms of steatorrhea, and prevent further episodes of acute pancreatitis by reducing thickened secretions and the obstruction of pancreatic ducts.

      Incorrect Answers: (A, B, D, E) Supplementation of these enzymes would not address the underlying cause of this patient’s symptoms. For more information, check out Med School Bootcamp’s videos on Pancreatic Pathology.
    10. Correct Answer: (B) This patient presenting with symptoms of cardiac obstruction (e.g., fainting) is found to have S3 and S4 heart sounds and a pedunculated intracardiac mass, findings that are pathognomonic for a cardiac myxoma. Cardiac myxomas are benign tumors most commonly found in the left atrium. They act as ball-valve obstructions that cause intermittent symptoms (e.g., dizziness, palpitations, syncope) and are the most common primary cardiac tumor in adults. Weight loss, fever, pallor, chills, fatigue, and other constitutional symptoms are also possible, as cardiac myxomas frequently secrete IL-6. Biopsy findings of scattered mesenchymal cells surrounded by gelatinous material (i.e., material composed of glycosaminoglycans) further support this diagnosis.

      Incorrect Answers: (A, C, D, E) These choices are incorrect because the most common location of cardiac myxomas is the left atrium. Note that the most common cardiac tumors in children are cardiac rhabdomyomas, which most commonly occur in the ventricles.
    11. Correct Answer: (D) This patient experienced symptoms of hypoglycemia (i.e., restlessness, sweating, confusion) due to over-supplementation of exogenous insulin. Mistakes in calculating the necessary insulin dosage after a high-carbohydrate meal are common. When adjusting insulin doses for a specific meal, the shorter-acting insulin dose alone should be adjusted, while the basal insulin dose is left the same. This patient’s father increased the doses of both insulin formulations around a high-carbohydrate meal, which most likely led to overcompensation of this child’s carbohydrate intake, causing a brief period of symptomatic hypoglycemia. Symptoms of hypoglycemia are multifactorial, including autonomic (e.g., sweating, restlessness) and neuroglycopenic (e.g., confusion) features.

      Incorrect Answers: (A, B, C, F) This patient’s glucose concentration of 72 mg/dL is not indicative of hyperglycemia. Symptoms classically associated with hyperglycemia include polyuria, polydipsia, polyphagia, and blurred vision (in severe cases). Increased glycogen metabolism would lead to hyperglycemia through the process of glycogenolysis. (E) Excessive renal glucose loss is typically associated with a gradual, chronic onset of symptoms. It would not cause acute symptoms such as restlessness, sweating, and confusion. (F) Increased glucagon secretion would promote hepatic glycogenolysis, which would lead to hyperglycemia, not hypoglycemia. (G & H) Nightmare disorder is characterized by distressing dreams that occur during REM sleep; sleep terror disorder is characterized by episodes of intense fear during non-REM sleep, with no memory of the event upon awakening.
    12. Correct Answer: (B) This elderly female has evidence of worsening osteoarthritis in her wrists and hands that causes her significant pain and impacts her daily functioning. The image of her hand reveals Heberden and Bouchard nodes (especially of the 3rd PIP), which are commonly seen in osteoarthritis. Osteoarthritis occurs due to age-related decreases in proteoglycan synthesis and chronic mechanical forces translated across joints. This process leads to the loss of cartilage elasticity and subsequent degeneration and inflammation of the cartilage. This can be seen as subchondral sclerosis, joint space narrowing, and the presence of osteophytes on imaging.

      Incorrect Answers: (A) Cortical thinning is associated with osteoporosis. Physical examination rarely reveals any changes until osteoporosis is advanced and leads to loss of height, kyphosis, or vertebral fractures. (C, D, E) Inflammation of digital tendons, neutrophilic infiltration of the synovium, and thickening of the synovium with pannus are associated with rheumatoid arthritis. This condition often manifests with joint pain and stiffness that affect multiple joints symmetrically. 
    13. Correct Answer: (C) This patient with a history of Parkinson's disease managed with Carbidopa-levodopa therapy is presenting with episodes of freezing (i.e. temporary, involuntary inability to move), unstable gait, and upper extremity rigidity. Patients with Parkinson's disease are commonly prescribed levodopa to increase dopamine levels in the central nervous system and carbidopa to decrease the peripheral conversion of levodopa (which can cross the blood-brain barrier) to dopamine (which cannot cross the blood-brain barrier) by DOPA decarboxylase. This combination is the classic treatment of choice for Parkinson's disease, but it is commonly affected by the on-off phenomenon. This refers to fluctuations in medication effectiveness, which cause patients to transition between periods of improved mobility (i.e., “on”) and the return of symptoms such as rigidity and tremors (i.e., “off”). This effect can be mitigated with a monoamine oxidase type B inhibitor (e.g., selegiline or rasagiline), which prevents the central conversion of dopamine into DOPAC (i.e., 3,4-dihydroxyphenylacetic acid), thereby prolonging the effect of the neurotransmitter.

      Incorrect Answers: (A, B, D, E) Levodopa is not associated with the inhibition of these enzymes. For additional information, check out Med School Bootcamp's video on Parkinson's Disease.
    14. Correct Answer: (D) This newborn is at risk for fetal alcohol syndrome (FAS) due to his mother’s alcohol intake. Signs and symptoms of FAS include developmental delay, microcephaly, smooth hypoplastic philtrum, down-slanting short palpebral fissures, epicanthal folds, heart defects (e.g., VSD), and many others.

      Incorrect Answers: (A) Hypospadias is a common congenital malformation that occurs due to impaired urethral folds and foreskin fusion, which causes incorrect positioning of the external urethral meatus. (B) Limb hypoplasia is seen in various syndromes and can occur due to drug use (e.g., thalidomide) or disorders of limb development (e.g., Potter sequence). (C) Neck webbing is classically associated with Turner syndrome. (E) Spasticity is commonly associated with disorders of the central nervous system (e.g., cerebral palsy). For additional information, check out Med School Bootcamp's video on Teratogenic Medications.
    15. Correct Answer: (E) This patient presents with delayed puberty, small penile and testicular size, the absence of pubic and facial hair, and bilateral gynecomastia. His physical exam findings, along with the elevated levels of follicle-stimulating hormone (FSH) and luteinizing hormone (LH) in the serum, suggest a primary gonadal failure due to Klinefelter syndrome (47,XXY).

      Klinefelter syndrome results from nondisjunction of the X chromosome during meiosis. Individuals with Klinefelter syndrome typically have small testes and hypogonadism; they may also present with delayed puberty and gynecomastia due to decreased testosterone and increased estrogen. The elevated FSH and LH levels in the serum indicate the gonadotropins’ efforts to stimulate the underdeveloped testes.

      Incorrect Answers: (A) Constitutional growth delay in puberty often occurs in children with delayed puberty due to the temporary halting of skeletal growth. Diagnostic evaluation will reveal decreased bone age in comparison to chronological age. This is in contrast with familial short stature, in which bone age is consistent with chronological age. (B) Congenital hypoplasia of hypothalamic gonadotropin-releasing hormone neurons causes GnRH deficiency and Kallman syndrome, which involves loss of olfactory senses (i.e., anosmia) and hypogonadotropic hypogonadism. (C) Inactivating mutation of the follicle-stimulating hormone receptor is a rare condition that manifests with poor ovarian function and infertility. FSH would likely be elevated due to loss of negative feedback inhibition, but this would not explain the alterations in testosterone and LH levels in this patient. (D) Mutations in the 21-hydroxylase gene typically manifest with ambiguous genitalia in females or salt-wasting in infancy.
    16. Correct Answer: (B) This patient with a history of chronic back pain has evidence of bone loss on MRI (e.g., reduced lumbar vertebral body height, bulging intervertebral discs) and spinal stenosis with evidence of impingement on physical examination (i.e., the “shopping cart” sign, which refers to the pain relief patients with spinal stenosis experience when in a flexed position, as if leaning over a shopping cart). Cigarette smoking is a major risk factor for bone loss and osteoporosis because it reduces blood flow to bones and inhibits the absorption of calcium, leading to decreased bone density. Other risk factors include malabsorption, malnutrition, anorexia, and certain medications (e.g., anticonvulsants, corticosteroids, PPIs, and anticoagulants).

      Incorrect Answers: (A) Excessive alcohol consumption is a risk factor, but this patient had not drunk in 5 years and only drank two beers per day when he did. (C, D, E) Diclofenac, loratadine, and opioids are not directly associated with the development of osteoporosis.
    17. Correct Answer: (B) This patient has a rapidly enlarging neck mass in the posterior triangle. Biopsy results reveal squamous epithelium with atypical lymphocytes and absent keratin formation, which are characteristic of nasopharyngeal carcinoma (NPC). NPC is endemic to southern China and other parts of Southeast Asia and Africa. Although this patient’s country of origin is not provided, his biopsy findings are consistent with NPC, which is most likely caused by an underlying Epstein-Barr virus (EBV) infection. EBV is a double-stranded DNA virus that is known to cause infectious mononucleosis and the development of multiple types of human cancer (e.g., nasopharyngeal carcinoma, lymphoma, gastric carcinoma). Other risk factors include smoking, heavy alcohol consumption, and ingestion of nitrosamine-containing food agents.

      Incorrect Answers: (A, C, D) These viruses are not directly related to the development of nasopharyngeal carcinoma. Cytomegalovirus is associated with mononucleosis and hepatitis; HIV is associated with the development of many opportunistic infections (e.g., toxoplasmosis or tuberculosis); human herpesvirus-8 often leads to Kaposi's sarcoma, a condition characterized by the generalized manifestation of initially painless, nonpruritic papules on the skin that develop into painful, submucosal plaques over time.
    18. Correct Answer: (B) This patient has poorly controlled type 2 diabetes mellitus. This is most likely due to her travel and work situation, which leads her to eat excessive amounts of fast food (i.e., overly processed, high-carbohydrate density foods) and forget to measure her blood glucose concentration. The best next action by the physician would be to further inquire about the barriers to diet adherence this patient is experiencing (e.g., financial ability, time requirement, food macronutrient knowledge). When physicians gain more information about their patients' barriers to obtaining healthy food, they can tailor interventions effectively and account for socioeconomic factors (e.g., food insecurity and accessibility).

      Incorrect Answers: (A) Creating an exercise regimen for a patient who already feels time constraints and cannot handle the current level of intervention is unlikely to be helpful. (C & D) Increasing this patient’s medications may be necessary, but it is unlikely to be effective if the patient’s dietary adherence is the primary issue. (E) Measuring the blood glucose concentration is unlikely to be helpful as it is more than likely elevated, especially with a known elevated hemoglobin A1c. Additionally, this patient displays no signs of HHS or DKA, so this is unwarranted. (F) Though monitoring of renal function is essential in patients with diabetes mellitus, this is unlikely to help improve this patient’s level of disease control. (G) Referral to a nutritionist could be helpful if a lack of macronutrient knowledge and healthy food choices were a source of this patient’s problem.
    19. Correct Answer: (E) A patient with a history of coronary artery disease presents to the emergency department with new-onset chest pain and ST elevations on ECG, suggesting an acute myocardial infarction (MI). Phenylephrine is a selective alpha-1 adrenergic agonist that primarily causes vasoconstriction. By constricting peripheral blood vessels, it increases systemic vascular resistance and, consequently, blood pressure. Phenylephrine has minimal effects on beta-adrenergic receptors and can cause reflex bradycardia due to the alpha-1-mediated increase in mean arterial pressure. As it does not increase heart rate, it is a suitable choice for a patient with an acute MI who needs blood pressure support without increasing cardiac workload.

      Incorrect Answers: (A, B, C, D) Dopamine, epinephrine, and isoproterenol may increase heart rate. Norepinephrine, while effective in increasing blood pressure, also has some beta-adrenergic effects that may lead to increased heart rate. Therefore, phenylephrine is the drug of choice in this scenario to avoid worsening myocardial ischemia. 
    20. Correct Answer: (E) This patient's presentation is suggestive of severe combined immunodeficiency (SCID), a group of disorders characterized by impaired T and B lymphocyte function leading to susceptibility to severe and recurrent viral, bacterial, fungal, and protozoal infections. In this case, the absence of T and B lymphocytes, along with the absence of characteristic V(D)J rearrangements in the bone marrow, suggests a defect in the recombination process necessary for the development of functional T and B cell receptors. RAG1 and RAG2 are responsible for the recombination of V(D)J gene segments, a critical step in the development of antigen receptors for both T and B lymphocytes. Mutations in RAG1 and RAG2 can lead to SCID and result in a lack of functional T and B cells, leaving the patient highly susceptible to infections, as seen in this case.

      Incorrect Answers: (A) Bruton tyrosine kinase deficiency (i.e., X-linked agammaglobulinemia) is associated with recurrent bacterial infections, particularly of the respiratory tract and ears, due to an inability to produce mature B cells and immunoglobulins. (B) CD40 ligand deficiency causes hyper IgM syndrome, which typically manifests with recurrent bacterial and opportunistic infections (e.g., Pneumocystis and CMV), particularly involving the respiratory tract and mucosal surfaces. This is an X-linked recessive condition and most often occurs in males. (C) Mevalonate kinase deficiency is associated with hyperimmunoglobulin-D syndrome (HIDS), a rare periodic fever syndrome that manifests with recurrent episodes of fever, lymphadenopathy, abdominal pain, arthralgia, and skin rashes. (D) NADPH oxidase deficiency (i.e., chronic granulomatous disease) leads to recurrent bacterial and fungal infections from catalase-positive organisms.
    21. Correct Answer: (D) This patient developed palpitations and heat intolerance after delivering a child. Exam findings include tachycardia, palmar sweating, lid lag, increased deep tendon reflexes, and an enlarged, nontender thyroid. Labs reveal TSH suppression and elevated T4/T3. This is most likely due to postpartum thyroiditis, a subtype of subacute lymphocytic thyroiditis that is due to follicular cell damage with lymphocytic infiltration, leading to the release of preformed thyroid hormone. This condition is typically self-limited, arises within a year of delivery, and begins with initial hyperthyroidism followed by a hypothyroid phase.

      Incorrect Answers: (A) Activation of the TSH receptors secondary to a mutation is a rare condition known as nonautoimmune hyperthyroidism. The onset of this patient’s symptoms following her pregnancy suggests postpartum thyroiditis as a more likely cause. (B) During pregnancy, thyroglobulin levels increase due to beta-human chorionic gonadotropin stimulation, but they typically decrease after birth. (C) Sheehan syndrome is caused by ischemic damage to the pituitary gland (not the hypothalamus) and may manifest with symptoms of hypothyroidism; this is not consistent with this patient’s presentation. (E) Graves disease occurs secondary to the autoimmune activation of the TSH receptors and causes symptoms of hyperthyroidism, including weight loss, tremors, palpitations, heat intolerance, and exophthalmos (i.e., bulging eyes). 
    22. Correct Answer: (A) This 53-year-old female presents with six years of stiffness and pain in her hands that cause difficulty when buttoning her clothes. Physical exam reveals multiple Swan neck deformities (i.e., PIP hyperextension, DIP flexion), which are representative of rheumatoid arthritis (RA). RA is associated with anti-citrullinated peptide antibodies (> 90% specificity), rheumatoid factor (less specific, can be elevated in infectious disease, other autoimmune diseases, and some healthy individuals), and antinuclear antibodies. RA is also associated with a genetic disposition for HLA-DR4 and HLA-DR1 proteins.

      Incorrect Answers: (B) Antimitochondrial antibodies are associated with primary biliary cholangitis, a chronic autoimmune liver disease characterized by progressive destruction of intrahepatic bile ducts that leads to symptoms such as pruritus, jaundice, and hepatomegaly. (C) Human leukocyte antigen-DQ2 antibodies are associated with celiac disease, which causes diarrhea, abdominal pain, bloating, and malabsorption. (D & E) Several conditions result from abnormalities in the precursors of erythroid and thrombopoietic cells (e.g., myelodysplastic syndromes), but these conditions are associated with hematologic abnormalities and would not explain this patient’s symptoms.
    23. Correct Answer: (E) A 73-year-old female with a 6-hour history of severe abdominal pain and a history of chronic atrial fibrillation is likely experiencing mesenteric ischemia due to an embolus originating from the heart. In cases of superior mesenteric artery (SMA) embolism, the blood supply to the affected segment of the intestine is compromised and can lead to acute mesenteric ischemia. The SMA supplies regions derived from the midgut, including structures from the distal duodenum to the proximal transverse colon. The blood flow from the superior mesenteric artery branches distally, so the proximal jejunum may initially appear normal due to collateral circulation from arterial arcades supplying this segment.

      Incorrect Answers: (A, B, C, D) The perfusion becomes progressively more compromised with distal movement along the intestine, leading to ischemic changes and infarction. Therefore, the proximal jejunum is spared compared to more distal portions of the small intestine. For more information about the anatomy and blood supply of the intestines, check out Med School Bootcamp’s videos on Abdominal Anatomy.
    24. Correct Answer: (A) This patient presents with signs and symptoms of cystic fibrosis, an autosomal recessive disease due to a defect in the CFTR gene on chromosome 7. This gene encodes the cystic fibrosis transmembrane conductance regulator (i.e., a chloride and bicarbonate channel responsible for fluid secretions and extracellular alkalinization). Patients with cystic fibrosis are unable to adequately secrete bicarbonate, leading to organ-specific manifestations (e.g., pancreatic insufficiency, dental caries, impaired gastrointestinal mucous secretion, cervical mucous thickening, and impaired fertility).

      Incorrect Answers: (B, C, D, E) Cystic fibrosis is not directly related to decreased secretion of these substances. For additional information, check out Med School Bootcamp's video on Cystic Fibrosis.
    25. Correct Answer: (E) This patient's symptoms of left lower quadrantanopia (i.e., loss of vision in the lower left quadrant of the visual field) and the inability to correctly identify tactile stimuli on the left side suggest a contralateral lesion in the right parietal lobe. The angular gyrus of the parietal lobe is a major component of the visual association cortex. Lesions of the right, or non-dominant, parietal lobe are associated with the development of hemispatial neglect syndrome (i.e., the inability to process sensory information on the contralateral side), as depicted by this patient.

      Incorrect Answers: (A & D) A lesion in the frontal lobe typically manifests as contralateral weakness or paralysis that affects the opposite side of the body, deficits related to executive functioning, and behavior changes. (B) A lesion in the left parietal lobe, which is most often the dominant hemisphere, manifests as Gerstmann syndrome, a condition associated with right-left confusion, difficulty writing (i.e., agraphia), and difficulty performing mathematical calculations (i.e., acalculia). (C & F) The temporal lobes are most notable for housing the primary auditory cortex and Wernicke area, which is responsible for language comprehension. A lesion in the left temporal lobe is most commonly associated with Wernicke aphasia, in which affected individuals demonstrate fluent but often incomprehensible speech. Lesions of the right temporal lobe are associated with impairments in interpreting and expressing fluctuations in vocal tone (i.e., aprosodia).
    26. Correct Answer: (C) This patient presents with signs of platelet dysfunction (e.g., nosebleeds, easy bruising, gingival bleeding). These symptoms, combined with the patient’s family history, normal platelet count, normal ristocetin cofactor response assay, and impaired thrombin-mediated platelet aggregation, are indicative of Glanzmann thrombasthenia, an aggregation disorder due to an autosomal recessive deficiency in platelet receptor GpIIb-IIIa, which leads to a defect in platelet plug formation. Most affected patients are asymptomatic or develop minor bleeding (e.g., petechiae, menorrhagia, epistaxis, purpura, gingival bleeding). Lab findings include increased bleeding time, normal platelets, and impaired platelet aggregation studies (i.e., impaired thrombin-mediated platelet aggregation). PT/INR, aPTT, D-dimer, and fibrin degradation products will all be within normal limits.

      Incorrect Answers: (A) Factor VII is activated by tissue thromboplastin to initiate the extrinsic pathway of the coagulation cascade. Deficiency would likely result in prolonged prothrombin time. (B) Fibrinogen is involved in the final steps of the common pathway of the coagulation cascade. Deficiency would likely show increased prothrombin time or activated partial thromboplastin time. Moreover, this patient’s normal response to ristocetin but impaired response to thrombin more likely indicates an issue in platelet functionality. (D) Platelet secretory granules, which contain von Willebrand factor and fibrinogen, are essential for platelet function. Defects of the granule storage pool may compromise the ability of platelets to respond appropriately to certain factors, including ristocetin; the isolated impairment of thrombin response seen in this patient’s laboratory studies more likely indicates dysfunctionality of glycoprotein IIb-IIIa. (E) Abnormalities in von Willebrand factor would demonstrate a compromised ability of platelets to respond to ristocetin, which is not seen in this patient. For additional information, check out Med School Bootcamp's video on Platelet Disorders.
    27. Correct Answer: (E) This patient has developed new-onset shortness of breath after living at a high altitude for 2 years. Exam findings include jugular venous distention and lower extremity pitting edema, which are most likely due to group 3 pulmonary hypertension from living at a high altitude. High altitude exposure is associated with decreased PiO2 (atmospheric oxygen); along with acute tachypnea and renal bicarbonate excretion, pulmonary vascular resistance increases in response to hypoxic pulmonary vasoconstriction.

      Incorrect Answers: (A, B, F) As the wall of the right ventricle thickens, diastolic compliance decreases. As a result, the height of the P waves in lead I increases (i.e., P pulmonale, right atrial enlargement), and the height of R waves in lead V1 increases (i.e., increased right ventricular mass). (C) Compensation to chronic hypoxia includes erythroid expansion (i.e., erythropoiesis), increased erythrocyte mass, and increased hematocrit. (D) Pulmonary vascular resistance increases due to high altitude exposure. Over time, this leads to pulmonary hypertension and eventual concentric right ventricular hypertrophy. For additional information, check out Med School Bootcamp's video on Pulmonary Hypertension.
    28. Correct Answer: (B) This 5-year-old female has developed dysentery (i.e., blood diarrhea), acute kidney injury (i.e., oliguria, elevated creatinine), anemia, and thrombocytopenia, representing hemolytic uremic syndrome (HUS). This is most likely due to infection with the O157:H7 serotype of enterohemorrhagic Escherichia coli. Gastrointestinal inflammation causes bloody diarrhea and facilitates the systemic circulation of bacterial toxins (e.g., Shiga-like toxin, verotoxin), which preferentially cause endothelial damage, especially in the glomerulus (due to Gb3 receptor expression). Once damaged, endothelial cells release cytokines that promote further damage, vasoconstriction, microthrombi formation, and microangiopathic hemolytic anemia (MAHA). This leads to end-organ ischemia and damage, most commonly in the kidneys because they receive a large percentage of total blood flow, and ultimately acute kidney injury (e.g., decreased GFR, azotemia).

      Incorrect Answers: (A, D, E) Campylobacter jejuni, Salmonella enterica serovar enteritidis, and Yersinia pestis gastrointestinal infections commonly cause symptoms such as diarrhea (often bloody), abdominal cramps, fever, and nausea, but they would not account for the renal symptoms associated with hemolytic uremic syndrome (HUS). (C) Rotavirus is the leading cause of gastroenteritis worldwide and causes symptoms such as severe watery diarrhea, vomiting, fever, and abdominal pain. It often leads to dehydration, particularly in young children.
    29. Correct Answer: (D) This patient has developed acute tubular necrosis (e.g., muddy brown casts, acute kidney injury, elevated creatinine) in the setting of Escherichia coli sepsis. This is most likely due to decreased renal blood flow caused by hypotension and renal ischemia. The areas of the kidney most likely to be affected by ischemia include the proximal convoluted tubule and the thick ascending limb due to greater levels of metabolic activity and oxygen consumption.

      Incorrect Answers: (A, B, C, E) These are not the locations most likely to be affected by ischemia in the setting of acute tubular necrosis. For more information, check out Med School Bootcamp’s video on Acute Tubular Necrosis.
    30. Correct Answer: (D) This 19-year-old male presents with bilateral parotid gland swelling, fever, malaise, and decreased appetite. This presentation is classic for an infection with Mumps virus, a highly contagious viral infection transmitted via airborne droplets, contaminated saliva, and/or respiratory secretions. Mumps affects the salivary glands (especially the parotid gland) before disseminating to the ovaries, testes, pancreas, central nervous system, and/or lacrimal, thyroid, and mammary glands. This can be differentiated from other causes of parotid gland swelling due to its bilateral nature, painful presentation, and fever. Sialadenitis is typically unilateral and painful, sialolithiasis is unilateral and partial with prandial pain, and salivary gland tumors are typically unilateral with systemic signs (e.g., weight loss, fatigue) and painless +/- signs of invasion (e.g., facial nerve compression and palsy).

      Incorrect Answers: (A) The Epstein-Barr virus is responsible for mononucleosis infections with characteristic findings of atypical lymphocytes on blood tests and symptoms such as fatigue, fever, sore throat, and hepatosplenomegaly. (B) Hepatitis B virus infection often leads to symptoms such as jaundice, fatigue, and abdominal pain. It is typically transmitted through contact with infected blood or bodily fluids (e.g., sexual contact, IV drug use). (C) The measles virus first causes symptoms such as high fever, cough, runny nose, and Koplik spots (i.e., small white spots with red borders) on the buccal mucosa, followed by a characteristic red, maculopapular, blotchy rash. (E) Rubella virus (i.e., German measles) is typically associated with a mild fever, sore throat, swollen lymph nodes, and a distinctive rash that starts on the face and spreads to the rest of the body.
    31. Correct Answer: (E) This patient with a past medical history of myasthenia gravis has developed increasing weakness, shortness of breath, and abdominal cramping. Further evaluation reveals bradycardia, tachypnea, weak cough, impaired inspiratory force, hyperactive bowel sounds, myogenic fasciculations, and decreased deep tendon reflexes. In the context of this patient’s medical and medication history, these symptoms are most likely due to an overdose of pyridostigmine, an acetylcholinesterase inhibitor used in patients with myasthenia gravis to improve synaptic ACh levels and decrease neuromuscular symptoms. In excess, this can lead to multiple cholinergic side effects, including bradycardia, hypotension, nausea, abdominal pain, diarrhea, hypoventilation, lethargy, weakness, paralysis, and bronchospasm.

      Incorrect Answers: (A) Aspiration pneumonia can occur in patients with myasthenia gravis due to weakness in the muscles involved in swallowing. It is often associated with fever, coughing, difficulty breathing, and the production of foul-smelling sputum. (B) Guillain-Barré syndrome is an immune-mediated polyneuropathy that leads to ascending symmetric muscle weakness, depressed deep tendon reflexes, and respiratory compromise. It frequently follows an infection, such as Campylobacter jejuni. (C) Insufficient dosing of prednisone in myasthenia gravis may lead to continued muscle weakness, but it would not account for the hyperactive bowel sounds and myogenic fasciculations seen in this patient. (D) Motor neuron diseases, such as amyotrophic lateral sclerosis (ALS), may cause similar symptoms (i.e., dysphagia and fasciculations); however, this patient’s medical and medication history better explains her respiratory and abdominal symptoms. For additional information, check out Med School Bootcamp's video on Myasthenia Gravis.
    32. Correct Answer: (B) Paracentric chromosomal inversions involve two breaks in one chromosome that create an intervening segment that does not contain the centromere. The segment is then inverted 180 degrees and reinserted. These inversions can survive for long periods (i.e., through multiple generations). Normally, during prophase I, each pair of chromosomes comes together and exchanges genetic material (i.e., crossing over). A single chromosome containing a paracentric inversion may present structural challenges to crossover events, requiring the chromosomes to form a loop and allowing the inverted segment of one chromosome to align its genes accordingly with the normal segment of the other corresponding chromosome. If a crossover event occurs within the formed loop, two disproportionate chromosomal segments are produced. One chromosome may contain two centromeres, whereas the other may contain none. If a fetus is conceived with either of these two deformed chromosomes, the pregnancy would be at risk of early spontaneous abortion. A liveborn child would likely be the result of a crossover event occurring outside of the loop, meaning two structurally similar chromosomal segments (i.e., not associated with aneuploidy) would have been produced.

      Incorrect Answers: (A, C, D, E) These answer choices are not reflective of the effect of paracentric inversions.
    33. Correct Answer: (F) This patient’s irritability, restlessness, tremors, palpitations, tachycardia, tachypnea, and hypertension are most likely due to recent amphetamine use. Amphetamines cause these symptoms via the increased synaptic release and inhibited reuptake of biogenic amines (e.g., epinephrine, norepinephrine, serotonin, and dopamine), which leads to sympathetic overstimulation and hallucinogenic effects.

      Incorrect Answers: (A, B, C, D, E) These answer choices are not directly associated with the mechanism of sympathetic activation that occurs in methamphetamine use. For additional information, check out Med School Bootcamp's video on Amphetamines
    34. Correct Answer: (A) This patient’s seizures, myopathy, peripheral neuropathy, and ataxic gait are most likely a result of Myoclonus epilepsy with ragged red fibers (MERRF). MERRF is an inherited mitochondrial disorder that is typically diagnosed in late childhood or adolescence and is characterized by progressive myoclonus and seizures, often associated with exercise. Other features include myopathy, cardiac arrhythmia, sensorineural hearing loss, short stature, optic atrophy, and dementia. The stained coarsely granular fibers (i.e., “ragged red fibers”) seen on muscle biopsy are pathognomonic for a diagnosis of MERRF. Because the underlying cause of this condition is mitochondrial dysfunction, ATP production will be affected.

      Incorrect Answers: (B, C, D, E) Genetic defects that affect the synthesis of these substances are not associated with this patient’s presentation or the presence of ragged red fibers on muscle biopsy evaluation.
    35. Correct Answer: (E) This patient's presentation of fatigue, marked pallor, and decreased proprioception and sensation to vibration in the lower extremities, along with laboratory findings of low hemoglobin, a high mean corpuscular volume (MCV), and a low reticulocyte count, suggests vitamin B12 deficiency anemia. The histopathology of the gastric mucosal biopsy showing atrophic gastritis with extensive lymphocyte infiltration suggests an autoimmune cause of vitamin B12 deficiency. In autoimmune atrophic gastritis, the body's immune system mistakenly targets and destroys parietal cells in the gastric mucosa. As indicated by the arrow pointing to “E,” these parietal cells are responsible for producing intrinsic factor, which is necessary for vitamin B12 absorption in the ileum. Therefore, serologic studies in this patient would most likely show autoantibodies against parietal cells or intrinsic factor that interfere with vitamin B12 absorption, leading to deficiency and subsequent anemia.

      Incorrect Answers: (A) This location contains epithelial pit cells (i.e., surface mucous cells) which primarily secrete mucus and protect the surface of the epithelium. (B) This layer is the submucosa, which holds vasculature, lymphatics, and Meissner’s plexus. (C) These structures are chief cells; they secrete pepsinogen, which aids in the breakdown of proteins in the stomach. (D) This structure is the muscularis mucosa, a thin layer of smooth muscle that aids in mechanical digestion.
    36. Correct Answer: (A) Amiodarone is a potent class III antiarrhythmic drug that is commonly used to treat atrial fibrillation and can effectively restore and maintain normal sinus rhythm. It increases the duration of the action potential and the end refractory period and can prolong the QT interval, which is observed in the ECG changes in this patient. However, amiodarone can also predispose patients to other cardiovascular effects, including heart failure, heart block, and fatal arrhythmias (e.g., torsades de pointes due to QT-interval prolongation), so it requires careful monitoring.

      Incorrect Answers: (B) Digoxin is a cardiac glycoside that manifests on ECG with a downsloping ST segment depression, a shortened QT interval, and a mildly prolonged PR interval. (C) Mexiletine is a class IB antiarrhythmic medication that typically shortens the QT interval and has minimal effect on the PR interval. (D) Phenytoin is an anticonvulsant medication that typically prolongs the PR interval and may also cause a widening of the QRS complex on an ECG. (E) Verapamil is a non-dihydropyridine calcium channel blocker that typically prolongs the PR interval and may cause slowing of the heart rate on ECG.
    37. Correct Answer: (E) This patient has developed neutropenic fever, a common oncologic emergency in patients receiving chemotherapy. This is caused by a severe decrease in absolute neutrophil count (i.e., ANC < 500 or expected to decrease to that level based on trends) that can lead to life-threatening infections. Though these infections can be severe (e.g., gram-negative rod bacteremia), patients often present with mild symptoms (e.g., low-grade fever, tachycardia, lack of erythema) due to lack of neutrophil-mediated inflammation. Patients should receive early empiric antibiotic therapy, and investigation for the source of the infection should be rapid and thorough (e.g., central line-associated bloodstream infection workup, blood cultures, imaging).

      Incorrect Answers: (A) Deficiency of terminal complement (C5-C9) is associated with an increased susceptibility to recurrent Neisseria infections, particularly meningococcal meningitis, due to impaired bacterial clearance. Symptoms often include fever, headaches, nuchal rigidity, and petechiae or hemorrhagic rash. (B) Hypogammaglobulinemia typically manifests with recurrent bacterial infections and an increased susceptibility to viral infections due to reduced levels of immunoglobulins. The onset of this patient’s symptoms 3 weeks after chemotherapy suggests this infection was more likely associated with the cancer treatments rather than the leukemia itself. (C) Impaired T-lymphocyte function is commonly seen in conditions such as DiGeorge syndrome, severe combined immunodeficiency (SCID), and acquired immunodeficiency syndrome (AIDS). (D) Inhibition of tumor necrosis factor α (TNF-α) function is most commonly achieved through medications such as infliximab, adalimumab, and etanercept, which are used in the treatment of various autoimmune and inflammatory conditions. These medications were not used in the treatment of this patient’s cancer. For additional information, check out Med School Bootcamp's video on Cell Cycle Dependent Drug Targets.
    38. Correct Answer: (A) This patient with a past medical history of acute lymphoblastic leukemia is seen for follow-up. The physician adds an agent to decrease the toxicity of high-dose methotrexate in his normal cells. This agent is most likely folinic acid (i.e., leucovorin or active folic acid), which bypasses the requirement for normal healthy cells to activate folic acid via dihydrofolate reductase (i.e., the enzymatic target of methotrexate). This “rescues” the normal cells from methotrexate’s adverse effects and is known as salvage therapy or leucovorin rescue therapy.

      Incorrect Answers: (B) Methionine synthase catalyzes the conversion of homocysteine into methionine. (C) Pyruvate decarboxylase is an enzyme that catalyzes the conversion of pyruvate into oxaloacetate in gluconeogenesis. (D) Thiamine pyrophosphate, the active form of vitamin B1, is a required cofactor for pyruvate dehydrogenase and is involved in carbohydrate metabolism via oxidative phosphorylation and the pentose phosphate pathway. (E) Thymidylate synthase is an enzyme crucial to DNA synthesis because it facilitates the production of thymidine; it is also the target of chemotherapeutic 5-fluorouracil. For additional information, check out Med School Bootcamp's video on Clinical Applications of Pyrimidine Pathway.
    39. Correct Answer: (D) This screening test is administered to 1,000 women with proven breast cancer and 1,000 women without breast cancer. Among those without breast cancer, 100 were falsely classified as positive. To find the expected number of false positives in a population of 100,000 women with a known prevalence rate of 80 cases per 100,000, the following calculation can be used:

      Expected number of false positives = (Number of people without breast cancer) x (Probability of a false positive) = (100000 - 80) x (100/1000) = 99,920 x 0.1 = 9,992

      Therefore, the expected number of false positives is 9,992.

      Incorrect Answers: (A, B, C, E)
      These answers are incorrect calculations of the false positive rate for this scenario. For additional information, check out Med School Bootcamp’s video on Diagnostic Tests.
    40. Correct Answer: (F) This elderly male developed dyspnea on exertion. Physical examination is unremarkable, and no significant murmurs are heard on auscultation at rest. This patient’s symptoms are most likely due to calcific aortic sclerosis (i.e., calcification and fibrosis of the aortic valve leaflets) due to age-related valvular damage. This is the most common cause of aortic stenosis, affecting 1 out of 3 people between the ages of 75 and 85. Symptoms (e.g., dyspnea on exertion, lightheadedness, dizziness) are caused by a relative inability to increase blood flow across the stenotic valve during periods of exertion. For additional information, check out Med School Bootcamp's video on Aortic Stenosis.

      Incorrect Answers: (A) Mitral regurgitation is described as a holosystolic murmur that is typically loudest over the apex with possible radiation to the axilla; it is associated with early rheumatic fever. (B) Mitral stenosis is a late rumbling diastolic murmur heard over the apex following an opening snap; it is associated with late rheumatic fever. (C) Severe aortic stenosis is associated with a loud late-peaking systolic murmur and a diminished or absent S2 due to higher left ventricular pressure needed to overcome immobile aortic cusps. It also may result in weak pulses (i.e., “pulsus parvus et tardus”) and radiation to the carotid arteries. (D) Tricuspid regurgitation is a holosystolic murmur best heard over the tricuspid area that increases with respiration and is associated with pulmonary hypertension and infective endocarditis. (E) Tricuspid stenosis is a diastolic murmur best heard over the tricuspid area.

    Studying for Step 1? Check out Med School Bootcamp, an all-in-one resource to make studying medicine easy.

    Save 100+ hours of your life studying with Bootcamp.com

    Get everything you need in one place. Start studying today for free.

    Start Studying for Free
    Hannah Brein, DAT Bootcamp Student